[obm-l] Probabilidades com Dados

2015-12-03 Por tôpico arkon
Algum bizu?
 
Em um jogo com três dados não-viciados, com faces representando números de 1 a 6, cada jogador deve fazer quantos arremessos seguidos quiser para chegar o mais próximo possível de um total de 21 pontos, sendo que a pontuação atribuída a um certo arremesso é igual à soma das pontuações mostradas nas faces superiores de cada um dos três dados. O jogador perde se a soma dos pontos acumulados nos diferentes arremessos ultrapassar 21 pontos. Com base nesssas informaçoes, qual a probabilidade de um jogador fazer exatamente 21 pontos em dois arremessos?--
Esta mensagem foi verificada pelo sistema de antivírus e 
 acredita-se estar livre de perigo.




Re: [obm-l] Probabilidades com Dados

2015-12-03 Por tôpico Mauricio de Araujo
Em 3 de dezembro de 2015 14:37, arkon  escreveu:

> Em um jogo com três dados não-viciados


​Acho que a abordagem deve ser mais ou menos assim:

pensa no primeiro arremesso... a soma dos pontos obtidos dos três dados
pode ser 3, 4, ..., 18. Ocorre que a probabilidade dessas somas não está
distribuida uniformemente, a chance de dar 3 não é a mesma de dar 10, por
exemplo...​ isso porque só tem um jeito de dar 3 que é sair 1 em cada dado,
priobabilidade de (1/6)^3. Para sair 10 temos várias outras maneiras de
isso ocorrer... uma maneira de achar isso seria fazer uma árvore e somar as
probablidades no final Depois vc pensa no segundo arremesso que passa a
ser dependente do resultado do primeiro... se o primeiro arremesso der soma
3 então o segundo tem de ser soma 18 e assim por diante...





-- 
Abraços

oɾnɐɹɐ ǝp oıɔıɹnɐɯ

-- 
Esta mensagem foi verificada pelo sistema de antiv�rus e
 acredita-se estar livre de perigo.



Re: [obm-l] Probabilidades com Dados

2015-12-03 Por tôpico Marcelo Salhab Brogliato
Acho que sai usando funções geradoras.

A resposta seria o coeficiente de x^21 da expansão (1/6x + 1/6x^2 + 1/6x^3
+ 1/6x^4 + 1/6x^5 + 1/6x^6)^6 = (x + x^2 + x^3 + x^4 + x^5 + x^6)^6 / 6^6.

Vejo alguns possíveis caminhos:

1) Veja que: x + x^2 + x^3 + x^4 + x^5 + x^6 = x(x^6 - 1) / (x-1).
Agora temos que dividir (x^6-1)^6 por (x-1)^6 e pegar o coeficiente de
x^(21-6) = x^15.
Usando Briot-Rufini, acho que dá para pegar esse coeficiente sem ter que
abrir tudo, mas não tenho certeza.

2) Seja p(x) = x + x^2 + x^3 + x^4 + x^5 + x^6 e f(x) = [p(x)]^6
Logo, podemos derivar f(x) 21 vezes e pegar o valor dele no ponto 0.

f'(x) = 6[p(x)]^5 p'(x)
f''(x) = 30[p(x)]^4 [p'(x)]^2 + 6[p(x)]^5 p''(x)
f'''(x) = ... (parece que vai dar trabalho demais)

3) Fazer na mão. Seja p(x) = x^6 + x^5 + x^4 + x^3 + x^2 + x^1 = (1, 1, 1,
1, 1, 1, 0). Assim: [p(x)]^2 = (1, 2, 3, 4, 5, 6, 5, 4, 3, 2, 1, 0, 0)
[p(x)]^4 = [p(x)]^2 * [p(x)]^2 = (1, 2, 3, 4, 5, 6, 5, 4, 3, 2, 1, 0, 0) *
(1, 2, 3, 4, 5, 6, 5, 4, 3, 2, 1, 0, 0)
Logo:
1*(1, 2, 3, 4, 5, 6, 5, 4, 3, 2, 1, 0, 0, 0, 0, 0, 0, 0, 0, 0, 0, 0, 0, 0,
0) +
2*(0, 1, 2, 3, 4, 5, 6, 5, 4, 3, 2, 1, 0, 0, 0, 0, 0, 0, 0, 0, 0, 0, 0, 0,
0) +
3*(0, 0, 1, 2, 3, 4, 5, 6, 5, 4, 3, 2, 1, 0, 0, 0, 0, 0, 0, 0, 0, 0, 0, 0,
0) +
4*(0, 0, 0, 1, 2, 3, 4, 5, 6, 5, 4, 3, 2, 1, 0, 0, 0, 0, 0, 0, 0, 0, 0, 0,
0) +
5*(0, 0, 0, 0, 1, 2, 3, 4, 5, 6, 5, 4, 3, 2, 1, 0, 0, 0, 0, 0, 0, 0, 0, 0,
0) +
6*(0, 0, 0, 0, 0, 1, 2, 3, 4, 5, 6, 5, 4, 3, 2, 1, 0, 0, 0, 0, 0, 0, 0, 0,
0) +
5*(0, 0, 0, 0, 0, 0, 1, 2, 3, 4, 5, 6, 5, 4, 3, 2, 1, 0, 0, 0, 0, 0, 0, 0,
0) +
4*(0, 0, 0, 0, 0, 0, 0, 1, 2, 3, 4, 5, 6, 5, 4, 3, 2, 1, 0, 0, 0, 0, 0, 0,
0) +
3*(0, 0, 0, 0, 0, 0, 0, 0, 1, 2, 3, 4, 5, 6, 5, 4, 3, 2, 1, 0, 0, 0, 0, 0,
0) +
2*(0, 0, 0, 0, 0, 0, 0, 0, 0, 1, 2, 3, 4, 5, 6, 5, 4, 3, 2, 1, 0, 0, 0, 0,
0) +
1*(0, 0, 0, 0, 0, 0, 0, 0, 0, 0, 1, 2, 3, 4, 5, 6, 5, 4, 3, 2, 1, 0, 0, 0,
0)

= (1, 4, 10, 20, 35, 56, 80, 104, 125, 140, 146, 140, 125, 104, 80, 56, 35,
20, 10, 4, 1, 0, 0, 0, 0)

Por fim, falta multiplicarmos de novo por [p(x)]^2, mas agora só queremos o
coeficiente x^21.

1*(1, 4, 10, 20, 35, 56, 80, 104, 125, 140, 146, 140, 125, 104, 80, 56, 35,
20, 10, 4, 1, 0, 0, 0, 0, 0, 0, 0, 0, 0, 0, 0, 0, 0, 0, 0, 0) +
2*(0, 1, 4, 10, 20, 35, 56, 80, 104, 125, 140, 146, 140, 125, 104, 80, 56,
35, 20, 10, 4, 1, 0, 0, 0, 0, 0, 0, 0, 0, 0, 0, 0, 0, 0, 0, 0) +
3*(0, 0, 1, 4, 10, 20, 35, 56, 80, 104, 125, 140, 146, 140, 125, 104, 80,
56, 35, 20, 10, 4, 1, 0, 0, 0, 0, 0, 0, 0, 0, 0, 0, 0, 0, 0, 0) +
:
:

Observando, vemos que o que queremos é: 56 + 2*80 + 3*104 + 4*125 + 5*140 +
6*146 + 5*140 + 4*125 + 3*104 + 2*80 + 1*56 = 4332.

Logo, a resposta é 4332 / 6^6 = 0,09284979 ~ 9,28%.

Abraços,
Marcelo

2015-12-03 14:37 GMT-02:00 arkon :

> Algum bizu?
>
> Em um jogo com três dados não-viciados, com faces representando números de
> 1 a 6, cada jogador deve fazer quantos arremessos seguidos quiser para
> chegar o mais próximo possível de um total de 21 pontos, sendo que a
> pontuação atribuída a um certo arremesso é igual à soma das pontuações
> mostradas nas faces superiores de cada um dos três dados. O jogador perde
> se a soma dos pontos acumulados nos diferentes arremessos ultrapassar 21
> pontos. Com base nesssas informaçoes, qual a probabilidade de um jogador
> fazer exatamente 21 pontos em dois arremessos?
>
> --
> Esta mensagem foi verificada pelo sistema de antivírus e
> acredita-se estar livre de perigo.

-- 
Esta mensagem foi verificada pelo sistema de antiv�rus e
 acredita-se estar livre de perigo.



Re: [obm-l] Probabilidades com Dados

2015-12-03 Por tôpico Marcelo Salhab Brogliato
Outro caminho seria usando recursão, mas seria uma tabela de 21x6. Não sei
o que daria mais trabalho, multiplicar os polinômios ou fazer a tabela, rs.

Abraços,
Marcelo

2015-12-03 18:43 GMT-02:00 Marcelo Salhab Brogliato :

> Acho que sai usando funções geradoras.
>
> A resposta seria o coeficiente de x^21 da expansão (1/6x + 1/6x^2 + 1/6x^3
> + 1/6x^4 + 1/6x^5 + 1/6x^6)^6 = (x + x^2 + x^3 + x^4 + x^5 + x^6)^6 / 6^6.
>
> Vejo alguns possíveis caminhos:
>
> 1) Veja que: x + x^2 + x^3 + x^4 + x^5 + x^6 = x(x^6 - 1) / (x-1).
> Agora temos que dividir (x^6-1)^6 por (x-1)^6 e pegar o coeficiente de
> x^(21-6) = x^15.
> Usando Briot-Rufini, acho que dá para pegar esse coeficiente sem ter que
> abrir tudo, mas não tenho certeza.
>
> 2) Seja p(x) = x + x^2 + x^3 + x^4 + x^5 + x^6 e f(x) = [p(x)]^6
> Logo, podemos derivar f(x) 21 vezes e pegar o valor dele no ponto 0.
>
> f'(x) = 6[p(x)]^5 p'(x)
> f''(x) = 30[p(x)]^4 [p'(x)]^2 + 6[p(x)]^5 p''(x)
> f'''(x) = ... (parece que vai dar trabalho demais)
>
> 3) Fazer na mão. Seja p(x) = x^6 + x^5 + x^4 + x^3 + x^2 + x^1 = (1, 1, 1,
> 1, 1, 1, 0). Assim: [p(x)]^2 = (1, 2, 3, 4, 5, 6, 5, 4, 3, 2, 1, 0, 0)
> [p(x)]^4 = [p(x)]^2 * [p(x)]^2 = (1, 2, 3, 4, 5, 6, 5, 4, 3, 2, 1, 0, 0) *
> (1, 2, 3, 4, 5, 6, 5, 4, 3, 2, 1, 0, 0)
> Logo:
> 1*(1, 2, 3, 4, 5, 6, 5, 4, 3, 2, 1, 0, 0, 0, 0, 0, 0, 0, 0, 0, 0, 0, 0, 0,
> 0) +
> 2*(0, 1, 2, 3, 4, 5, 6, 5, 4, 3, 2, 1, 0, 0, 0, 0, 0, 0, 0, 0, 0, 0, 0, 0,
> 0) +
> 3*(0, 0, 1, 2, 3, 4, 5, 6, 5, 4, 3, 2, 1, 0, 0, 0, 0, 0, 0, 0, 0, 0, 0, 0,
> 0) +
> 4*(0, 0, 0, 1, 2, 3, 4, 5, 6, 5, 4, 3, 2, 1, 0, 0, 0, 0, 0, 0, 0, 0, 0, 0,
> 0) +
> 5*(0, 0, 0, 0, 1, 2, 3, 4, 5, 6, 5, 4, 3, 2, 1, 0, 0, 0, 0, 0, 0, 0, 0, 0,
> 0) +
> 6*(0, 0, 0, 0, 0, 1, 2, 3, 4, 5, 6, 5, 4, 3, 2, 1, 0, 0, 0, 0, 0, 0, 0, 0,
> 0) +
> 5*(0, 0, 0, 0, 0, 0, 1, 2, 3, 4, 5, 6, 5, 4, 3, 2, 1, 0, 0, 0, 0, 0, 0, 0,
> 0) +
> 4*(0, 0, 0, 0, 0, 0, 0, 1, 2, 3, 4, 5, 6, 5, 4, 3, 2, 1, 0, 0, 0, 0, 0, 0,
> 0) +
> 3*(0, 0, 0, 0, 0, 0, 0, 0, 1, 2, 3, 4, 5, 6, 5, 4, 3, 2, 1, 0, 0, 0, 0, 0,
> 0) +
> 2*(0, 0, 0, 0, 0, 0, 0, 0, 0, 1, 2, 3, 4, 5, 6, 5, 4, 3, 2, 1, 0, 0, 0, 0,
> 0) +
> 1*(0, 0, 0, 0, 0, 0, 0, 0, 0, 0, 1, 2, 3, 4, 5, 6, 5, 4, 3, 2, 1, 0, 0, 0,
> 0)
>
> = (1, 4, 10, 20, 35, 56, 80, 104, 125, 140, 146, 140, 125, 104, 80, 56,
> 35, 20, 10, 4, 1, 0, 0, 0, 0)
>
> Por fim, falta multiplicarmos de novo por [p(x)]^2, mas agora só queremos
> o coeficiente x^21.
>
> 1*(1, 4, 10, 20, 35, 56, 80, 104, 125, 140, 146, 140, 125, 104, 80, 56,
> 35, 20, 10, 4, 1, 0, 0, 0, 0, 0, 0, 0, 0, 0, 0, 0, 0, 0, 0, 0, 0) +
> 2*(0, 1, 4, 10, 20, 35, 56, 80, 104, 125, 140, 146, 140, 125, 104, 80, 56,
> 35, 20, 10, 4, 1, 0, 0, 0, 0, 0, 0, 0, 0, 0, 0, 0, 0, 0, 0, 0) +
> 3*(0, 0, 1, 4, 10, 20, 35, 56, 80, 104, 125, 140, 146, 140, 125, 104, 80,
> 56, 35, 20, 10, 4, 1, 0, 0, 0, 0, 0, 0, 0, 0, 0, 0, 0, 0, 0, 0) +
> :
> :
>
> Observando, vemos que o que queremos é: 56 + 2*80 + 3*104 + 4*125 + 5*140
> + 6*146 + 5*140 + 4*125 + 3*104 + 2*80 + 1*56 = 4332.
>
> Logo, a resposta é 4332 / 6^6 = 0,09284979 ~ 9,28%.
>
> Abraços,
> Marcelo
>
> 2015-12-03 14:37 GMT-02:00 arkon :
>
>> Algum bizu?
>>
>> Em um jogo com três dados não-viciados, com faces representando números
>> de 1 a 6, cada jogador deve fazer quantos arremessos seguidos quiser para
>> chegar o mais próximo possível de um total de 21 pontos, sendo que a
>> pontuação atribuída a um certo arremesso é igual à soma das pontuações
>> mostradas nas faces superiores de cada um dos três dados. O jogador perde
>> se a soma dos pontos acumulados nos diferentes arremessos ultrapassar 21
>> pontos. Com base nesssas informaçoes, qual a probabilidade de um jogador
>> fazer exatamente 21 pontos em dois arremessos?
>>
>> --
>> Esta mensagem foi verificada pelo sistema de antivírus e
>> acredita-se estar livre de perigo.
>
>
>

-- 
Esta mensagem foi verificada pelo sistema de antiv�rus e
 acredita-se estar livre de perigo.



Re: [obm-l] Probabilidades com Dados

2015-12-03 Por tôpico Bernardo Freitas Paulo da Costa
2015-12-03 18:43 GMT-02:00 Marcelo Salhab Brogliato :
> Acho que sai usando funções geradoras.
Sempre sai, principalmente com um computador para fazer as contas ;-)

> A resposta seria o coeficiente de x^21 da expansão (1/6x + 1/6x^2 + 1/6x^3 +
> 1/6x^4 + 1/6x^5 + 1/6x^6)^6 = (x + x^2 + x^3 + x^4 + x^5 + x^6)^6 / 6^6.
>
> Vejo alguns possíveis caminhos:
>
> 1) Veja que: x + x^2 + x^3 + x^4 + x^5 + x^6 = x(x^6 - 1) / (x-1).
> Agora temos que dividir (x^6-1)^6 por (x-1)^6 e pegar o coeficiente de
> x^(21-6) = x^15.
> Usando Briot-Rufini, acho que dá para pegar esse coeficiente sem ter que
> abrir tudo, mas não tenho certeza.

Tem um jeito mais fácil depois que você viu isso. Temos que achar o
coeficiente de x^21 em [ x(x^6 - 1)/(x - 1) ]^6, que é, como você
disse, o coeficiente de x^15 em [ (x^6 - 1)/(x - 1) ]^6. Que é o
coeficiente de x^15 em (x^6 - 1)^6 / (x - 1)^6 = (1 - x^6)^6 / (1 -
x)^6. O numerador tem apenas três termos que contribuem, a saber:
1, -6x^6, binom(6,2)x^12

Isso quer dizer que temos que achar os coeficientes de x^15, x^9 e x^3
na expansão de (1 - x)^{-6}. Ora, pelo teorema binomial, é
simplesmente:
binom(-6,15)(-1)^15, binom(-6,9)(-1)^9, binom(-6,3)(-1)^3. Agora, note
que (-6,k) = (-6)(-6-1) ... (-6-k+1)/k! = (-1)^k * binom(6+k-1,k) =
(-1)^k binom(5+k,k) = (-1)^k binom(5+k,5).

Pronto, agora é só multiplicar e somar:
binom(20,5) - 6*binom(14,5) + binom(6,2)*binom(8,5) = 15504 - 6*2002 +
15*56 = 4332.

Ah, sim, não esqueça de dividir por 6^6.
-- 
Bernardo Freitas Paulo da Costa

-- 
Esta mensagem foi verificada pelo sistema de antiv�rus e
 acredita-se estar livre de perigo.


=
Instru��es para entrar na lista, sair da lista e usar a lista em
http://www.mat.puc-rio.br/~obmlistas/obm-l.html
=


[obm-l] Probabilidades e inferencia estatistica

2011-05-02 Por tôpico mathematica


Ola,
Preciso da vossa ajuda amigos:

Seja um ponto activo num eixo numerico, que se move desde a origem em  
direcao positiva com propbabilidade p, move-se na direcao negativa com  
probabilidade 1-p. Ao mover-se n vezes, seja X o tempo que se move o  
ponto em direcao positiva, e Y a posicao do ponto.

1. Calcule a distribuicao de probabilidade de X e Y.
2. Calcule a funcao geradora de momento de X e Y.
3. Use 2. para calcular o valor esperado e a variancia de Y.

1- Assume-se que o tamanho de um produto obedece a distribuicao normal  
(mui, sigma ao quadrado). Se o valor populacional deste produto for  
maior que 7.0. O resultado obtido a partir de 16 amostras, i.e, x1,  
x2, ...,x16, o somatorio de Xi (i avaliado desde 1 a 16) = 113.6 e  
somatorio de Xi ao quadrado = 808.96. Use a tabela abaixo.
a) Pretende-se testar se o valor significativo populacional do tamanho  
deste producto e maior que 7.0. analise a hipotese apropriada e teste  
com o nivel de significancia alfa = 5%.
b) Calcule o intervalo de confianca em 95% da variancia populacional  
deste produto.


Rene

ABAIXO PODE VER-SE O EXERCICIO NO ORIGINAL A PARTIR DO INGLES PARA  
MAIOR COMPREENSAO POR CAUSA DA TRADUCAO.



2 [probability  statistics]
1. There is a active point in number line of 1-Dimension. It starts  
from origin, and it moves toward positive direction with probability  
p, moves toward negative direction with probability 1-p, one by one.  
When it moved n times, let X be the time that how many times did the  
point move toward positive direction, and let Y be the point(place) of  
the point.


(1). Calculate the probability distribution of X and Y.
(2). Calculate the moment-generating function of X and Y.
(3). Using (2), calculate the expectation and variance of Y.


2. Assume that the size of some product follows the normal  
distribution . We think if population mean value of this product is  
longer than 7.0. Given that  measured  result of 16 samples is  ,
and  . You can use following table.


(1). We would like to test if population mean value of this products  
size is longer than 7.0. Make appropriate hypothesis, and test with  
significance level  .
(2). Calculate the 95% confidence interval of population variance of  
this product.



=
Instruções para entrar na lista, sair da lista e usar a lista em
http://www.mat.puc-rio.br/~obmlistas/obm-l.html
=


[obm-l] PROBABILIDADES INSIDIOSAS!

2010-04-16 Por tôpico Jorge Luis Rodrigues e Silva Luis

Ok! Adalberto e demais colegas! É o que podemos chamar de problemas 
contra-intuitivos...

 

Vamos supor que a faculdade tenha uma taxa de matrículas de 50/90, ou 
aproximadamente 56% para mulheres em comparação com 60/100 ou 60% para homens, 
e tenha dois departamentos. No departamento 1, 50 mulheres inscrevem-se e 20 
são aceitas; 30 homens inscrevem-se e 10 são aceitos. A proporção de matrículas 
de 20/50, ou 40%, é comparativamente favorável às mulheres em relação à 
proporção de matrícula de homens, 10/30, aproximadamente 33%. No departamento 
2, 40 mulheres inscrevem-se e 30 são aceitas; 70 homens inscrevem-se e 50 são 
aceitos. A proporção de matrículas de mulheres é 30/40, ou 75%, comparada com a 
proporção de matrículas de homens de 50/70, ou 71%. Apesar disso, quando os 
dois conjuntos de estatísticas são combinados, a proporção de matrícula de 
mulheres, 50/90, é menor que a de homens, 60/100. Afinal! Se cada departamento 
é contado apenas uma vez e não há sobreposição, como é possível ter uma 
proporção maior de mulheres em cada departameto e uma proporção menor no 
conjunto total?

 

Dentre os prisioneiros A, B e C o juiz decidiu livrar a pele de um dos 
condenados. Ele diz ao prisioneiro A: Joguei aqui meu dado perfeitamente 
aleatório, e com ele já decidi quem de vocês será liberado. Não posso dizer 
ainda se vai ser você ou não, mas vou lhe contar um segredo: o prisioneiro B 
precisa se preparar porque vai curtir cadeia pelo resto de seus dias. Agora, 
se você A, quiser, pode trocar de destino com o prisioneiro C. Pense nisso. 
Mas então? Vale a pena trocar? Será esta uma resposta intuitiva? 
Contra-intuitiva? Quais as chances de que isso realmente aconteça?

 

A probabilidade de que um computador dê defeito após ele ter sido usado K vezes 
é G(k). Qual a probabilidade da máquina estar quebrada após N usos consecutivos 
se durante as M operações anteriores ela estava funcionando? (Probleminha 
esquisito!)

 

Abraços e Divirtam-se!
  
_
Você sabia que seu navegador te ajuda a ficar longe de vírus? Leia mais sobre 
isso.
http://www.microsoft.com/brasil/windows/internet-explorer/?WT.mc_id=1500

[obm-l] Re: [obm-l] RES: [obm-l] Re: [obm-l] Probabilidades Geométricas

2010-02-05 Por tôpico Adalberto Dornelles
Olá Albert

 Bem, quando propus o problema, já adiantei que a resposta (numérica) era
 igual a 9,31% - veja minha mensagem original abaixo.

Pensei que se tratava da resposta ao problema do círculo. Achei
curioso o resultado ser o mesmo.

 Entretanto o desafio é resolver o problema analiticamente, para, depois,
 atacar o 2º problema - este, sim, é bastante difícil.

Ah... sim, eu sei. É que não tenho muito talento para resoluções analíticas.

Abraço,
Adalberto

=
Instruções para entrar na lista, sair da lista e usar a lista em
http://www.mat.puc-rio.br/~obmlistas/obm-l.html
=


[obm-l] Re: [obm-l] Probabilidades Geométricas

2010-02-04 Por tôpico Adalberto Dornelles
Olá,

Em 19 de janeiro de 2010 13:18, Albert Bouskela bousk...@msn.com escreveu:
 1º Problema:
 Considere um triângulo equilátero. Calcule a probabilidade de um segmento de
 reta, determinado por um ponto qualquer de um dos lados desse triângulo e
 por outro ponto qualquer de um dos outros dois lados adjacentes, ser maior
 que a altura do triângulo.

Algo entre 9,28% e 9,43%?
Usei Monte Carlo...

Abraço,
Adalberto

=
Instruções para entrar na lista, sair da lista e usar a lista em
http://www.mat.puc-rio.br/~obmlistas/obm-l.html
=


[obm-l] Re: [obm-l] Probabilidades Geométricas

2010-02-04 Por tôpico Adalberto Dornelles
Olá


Em 13 de janeiro de 2010 18:23, Albert Bouskela bousk...@msn.com escreveu:
 1º Problema:
 Considere um triângulo equilátero. Calcule a probabilidade de um segmento de
 reta, determinado por um ponto qualquer de um dos lados desse triângulo e
 por outro ponto qualquer de um dos outros dois lados adjacentes, ser maior
 que a altura do triângulo.

Algo entre 9,28% e 9,43%?
Usei Monte Carlo...

Abraço,
Adalberto

=
Instruções para entrar na lista, sair da lista e usar a lista em
http://www.mat.puc-rio.br/~obmlistas/obm-l.html
=


[obm-l] RES: [obm-l] Re: [obm-l] Probabilidades Geométri cas

2010-02-04 Por tôpico Albert Bouskela
Olá!

Bem, quando propus o problema, já adiantei que a resposta (numérica) era
igual a 9,31% - veja minha mensagem original abaixo.

Uma aproximação melhor é 9,310031788%.

Logo, a aproximação que você encontrou, usando Monte Carlo, é bastante
razoável. Entretanto...

Entretanto o desafio é resolver o problema analiticamente, para, depois,
atacar o 2º problema - este, sim, é bastante difícil.

Saudações,
AB

1º Problema:
 
Considere um triângulo equilátero. Calcule a probabilidade de um segmento de
reta, determinado por um ponto qualquer de um dos lados desse triângulo e
por outro ponto qualquer de um dos outros dois lados adjacentes, ser maior
que a altura do triângulo.
 
Notas:
 
1)Paradoxo de Bertrand (Bertrand's Paradox): “Given a circle. Find
the probability that a chord chosen at random be longer than the side of an
inscribed equilateral triangle”.
2)Referência na Internet: http://www.cut-the-knot.com/bertrand.html
 
Para os curiosos, a resposta (numérica) é 9,31%.
 
 
2º Problema:
 
Considere uma caixa de base quadrada, cujos lados (da base) são unitários.
Na base desta caixa, são traçados dois segmentos de reta:
 
1)A própria diagonal da base; e
2)O segmento de reta entre os pontos médios de dois lados opostos.
 
Toma-se uma agulha de comprimento também unitário e joga-se, aleatoriamente,
dentro da caixa. Pergunta-se:
 
Qual é a probabilidade da agulha, então pousada horizontalmente na base da
caixa (por hipótese!), interceptar (em um ponto qualquer) o segmento de reta
de número “1”, descrito acima? E o de número “2”?
 
Vejam um problema análogo (mas muito mais fácil) em: 
http://www.cut-the-knot.com/fta/Buffon/buffon9.html

Albert Bouskela
bousk...@msn.com

 -Mensagem original-
 De: owner-ob...@mat.puc-rio.br [mailto:owner-ob...@mat.puc-rio.br] Em
 nome de Adalberto Dornelles
 Enviada em: quinta-feira, 4 de fevereiro de 2010 17:53
 Para: obm-l@mat.puc-rio.br
 Assunto: [obm-l] Re: [obm-l] Probabilidades Geométricas
 
 Olá
 
 
 Em 13 de janeiro de 2010 18:23, Albert Bouskela bousk...@msn.com
 escreveu:
  1º Problema:
  Considere um triângulo equilátero. Calcule a probabilidade de um
segmento
 de
  reta, determinado por um ponto qualquer de um dos lados desse triângulo
e
  por outro ponto qualquer de um dos outros dois lados adjacentes, ser
maior
  que a altura do triângulo.
 
 Algo entre 9,28% e 9,43%?
 Usei Monte Carlo...
 
 Abraço,
 Adalberto
 
 ===
 ==
 Instruções para entrar na lista, sair da lista e usar a lista em
 http://www.mat.puc-rio.br/~obmlistas/obm-l.html
 ===
 ==


=
Instruções para entrar na lista, sair da lista e usar a lista em
http://www.mat.puc-rio.br/~obmlistas/obm-l.html
=


[obm-l] Probabilidades Geométricas

2010-01-13 Por tôpico Albert Bouskela
Olá!

 

Já que todos estão falando sobre Probabilidades Geométricas, há tempos
atrás, pensei em dois problemas. O primeiro é bastante difícil, um pouco
trabalhoso, entretanto é possível resolvê-lo. Já o segundo é digno de um
Buffon. Lá vão eles:

 

1º Problema:

 

Considere um triângulo equilátero. Calcule a probabilidade de um segmento de
reta, determinado por um ponto qualquer de um dos lados desse triângulo e
por outro ponto qualquer de um dos outros dois lados adjacentes, ser maior
que a altura do triângulo.

 

Notas:

 

1)Paradoxo de Bertrand (Bertrand's Paradox): “Given a circle. Find
the probability that a chord chosen at random be longer than the side of an
inscribed equilateral triangle”.

2)Referência na Internet: http://www.cut-the-knot.com/bertrand.html

 

Para os curiosos, a resposta (numérica) é 9,31%.

 

 

2º Problema:

 

Considere uma caixa de base quadrada, cujos lados (da base) são unitários.
Na base desta caixa, são traçados dois segmentos de reta:

 

1)A própria diagonal da base; e

2)O segmento de reta entre os pontos médios de dois lados opostos.

 

Toma-se uma agulha de comprimento também unitário e joga-se, aleatoriamente,
dentro da caixa. Pergunta-se:

 

Qual é a probabilidade da agulha, então pousada horizontalmente na base da
caixa (por hipótese!), interceptar (em um ponto qualquer) o segmento de reta
de número “1”, descrito acima? E o de número “2”?

 

Vejam um problema análogo (mas muito mais fácil) em:
http://www.cut-the-knot.com/fta/Buffon/buffon9.html

 

Ralph, você vai se aventurar no 2º problema?

 

Saudações a todos,

AB



Re: [obm-l] probabilidades

2009-10-25 Por tôpico Silas Gruta
Mestre Ralph,

obrigado pela excelente explicação. Realmente, entre as alternativas havia o
n° 7 e o n° 8. Serviu o 8.

grande abraço

Silas



2009/10/23 Ralph Teixeira ralp...@gmail.com

 Suponha que são m meias vermelhas de um total de N meias.

 A probabilidade da 1a meia ser vermelha é m/N.

 Dada que a 1a é vermelha, a probabilidade da segunda ser vermelha
 (pelo enunciado, acho natural supor que é sem reposição, já que as
 duas meias sao tiradas ao mesmo tempo) é (m-1)/(N-1).

 Então a probabilidade de ambas serem vermelhas é m(m-1)/(N(N-1)), isto é:

 14m(m-1)=5N(N-1)

 Não há solução única, mas note que N tem que ser (múltiplo de 7), ou
 (múltiplo de 7, mais um). Se você tiver opções, veja qual serve (na
 pior hipótese, veja quais dão raízes inteiras para m naquela
 quadrática).

 Abraço, Ralph

 2009/10/22 Silas Gruta silasgr...@gmail.com:
  Boa noite, colegas
  Poderiam fazer a gentileza de explicar-me como se resolve a seguinte
  questão, fiquei bem confuso:
  Uma gaveta contém meias. Retirando-se duas meias ao acaso, a
 probabilidade
  de que as meias sejam ambas vermelhas é de 5/14. Qual dos números a
 seguir
  pode expressar a quantidade de meias na gaveta?
 
  Agradeço a ajuda
  --
  Silas Gruta
 

 =
 Instruções para entrar na lista, sair da lista e usar a lista em
 http://www.mat.puc-rio.br/~obmlistas/obm-l.htmlhttp://www.mat.puc-rio.br/%7Eobmlistas/obm-l.html
 =




-- 
Silas Gruta


Re: [obm-l] probabilidades

2009-10-23 Por tôpico Ralph Teixeira
Suponha que são m meias vermelhas de um total de N meias.

A probabilidade da 1a meia ser vermelha é m/N.

Dada que a 1a é vermelha, a probabilidade da segunda ser vermelha
(pelo enunciado, acho natural supor que é sem reposição, já que as
duas meias sao tiradas ao mesmo tempo) é (m-1)/(N-1).

Então a probabilidade de ambas serem vermelhas é m(m-1)/(N(N-1)), isto é:

14m(m-1)=5N(N-1)

Não há solução única, mas note que N tem que ser (múltiplo de 7), ou
(múltiplo de 7, mais um). Se você tiver opções, veja qual serve (na
pior hipótese, veja quais dão raízes inteiras para m naquela
quadrática).

Abraço, Ralph

2009/10/22 Silas Gruta silasgr...@gmail.com:
 Boa noite, colegas
 Poderiam fazer a gentileza de explicar-me como se resolve a seguinte
 questão, fiquei bem confuso:
 Uma gaveta contém meias. Retirando-se duas meias ao acaso, a probabilidade
 de que as meias sejam ambas vermelhas é de 5/14. Qual dos números a seguir
 pode expressar a quantidade de meias na gaveta?

 Agradeço a ajuda
 --
 Silas Gruta


=
Instruções para entrar na lista, sair da lista e usar a lista em
http://www.mat.puc-rio.br/~obmlistas/obm-l.html
=


[obm-l] probabilidades

2009-10-22 Por tôpico Silas Gruta
Boa noite, colegas

Poderiam fazer a gentileza de explicar-me como se resolve a seguinte
questão, fiquei bem confuso:

Uma gaveta contém meias. Retirando-se duas meias ao acaso, a probabilidade
de que as meias sejam ambas vermelhas é de 5/14. Qual dos números a seguir
pode expressar a quantidade de meias na gaveta?

Agradeço a ajuda

-- 
Silas Gruta


[obm-l] Re: [obm-l] Re: [obm-l] probabilidades caça-niquel

2009-05-10 Por tôpico Palmerim Soares
Olá Thelio e Mestre Ralph
Muito pertinentes os comentários do Ralph. Realmente, deve haver extremo
esmero na formulação dos enunciados, sobretudo quando o assunto é
probabilidades, que costuma causar confusão justamente por causa das
ambiguidades nos enunciados.

Bem, não custa, agora, apresentar uma solução diferente para esta questão:

Exibir um resultado é uma tarefa que pode ser realizada em 3 etapas
sucessivas:
1ª Etapa) Escolha das 3 frutas que irão comparecer no resultado: C(6,3)=20;
2ª Etapa) Escolha da fruta que comparecerá 2 vezes no resultado: C(3,1)=3;
3ª Etapa) Escolha das 2 posições destinadas às frutas desiguais: A(4,2)=12
Pelo Princípio Fundamental da Contagem há: 20 X 3 X 12 = 720 possíveis
resultados, e, como escreveu o Ralph, já que eles são todos igualmente
prováveis, a probabilidade pedida
é 720/1296=5/9.
Vale notar que a 3ª etapa poderia ser: posicionar as 4 frutas, ou seja,
fazer uma permutação de 4 elementos, sendo 2 repetidos, o que equivaleria
exatamente a calcular os anagramas da palavra BALA. (B de Banana, A de
Amora, L de Laranja e A de Ameixa)
Então (só para o Thelio ter uma visão geral) o que se quer nesse problema,
em última análise, é fazer uma permutação de 4 elementos, sendo que dois
deles são iguais entre si. Mas antes de fazer essa permutação com elementos
repetidos, precisamos escolher as frutas, o que foi feito nas etapas 1 e 2.

Finalmente, acho que um enunciado que seria aprovado pelo Mestre Ralph seria
assim:
(corrija-me se estiver errado, mestre, porque quero aplicar essa questão em
um simulado)

Numa máquina de caça-níquel, cada resultado é formado por 4 quaisquer de 6
frutas diferentes. Supondo que um resultado pode apresentar frutas
repetidas, calcule a probabilidade de um resultado apresentar duas frutas
iguais e duas outras frutas diferentes entre si.

Abraços,
Palmerim


2009/5/8 Ralph Teixeira ralp...@gmail.com

 Oi, Thelio.

 Vamos fazer as seguintes hipóteses:

 a) O resultado é formado por 4 símbolos; (isto está bem explícito em
 4 quaisquer...)
 b) Cada símbolo pode ser uma de seis frutas, que designarei por A, B,
 C, D, E, F (também razoavelmente explícito em de 6 frutas
 diferentes...);
 c) Um resultado pode apresentar símbolos iguais (por exemplo, pode ser
 AADE) -- isto está dito, mas com um português ligeiramente ambíguo;
 digo isso pois **gramaticalmente** podendo haver repetição poderia
 se referir a 4 símbolos ou a 6 frutas... mas faz mais sentido se
 for 4 símbolos, podendo haver repetição, que é a minha
 interpretação; a outra interpretação, 6 frutas diferentes, podendo
 haver repetição é meio contraditória...
 d) Em cada símbolo, cada fruta tem a mesma probabilidade de aparecer
 (razoável, mas não é nem um pouco óbvio; aliás, só vou supor isso
 porque tenho que resolver o problema e ele não indicou as
 probabilidades de cada fruta; num caça-níqueis de verdade, isto não
 costuma ser verdadeiro);
 e) Os 4 símbolos são independentes entre si, isto é, o símbolo que
 aparece na primeira janela não afeta de maneira alguma o símbolo da
 segunda (bem razoável, mas também não é certo no caso geral).
 f) O que o enunciado quer é a probabilidade de aparecerem 3 frutas
 distintas, sendo uma delas repetida (se eu quisesse ser muito muito
 chato, diria que AABB tem duas frutas AA iguais e duas frutas BB
 desiguais **da primeira** -- não acho que era isso que o enunciado
 tinha em mente, acho que eles querem dizer, duas frutas iguais e
 duas OUTRAS, desiguais ENTRE SI.). Em linguagem de pôquer: qual é a
 chance de dar um par?

 Agora sim, com tudo destrinchado, eu consigo resolver o problema. Há
 6.6.6.6=1296 possíveis resultados, todos igualmente prováveis graças a
 (d) e (e). Quantos são da forma XXYZ (ou permutações)?
 i) Primeiro, vou escolher as frutas que vão aparecer na minha
 sequencia: note que X é bem distinto de Y e Z, que são intercambiáveis
 neste momento. Há 6 maneiras de escolher X; agora, há C(5,2) maneiras
 de escolher as frutas Y e Z. Então há 6.C(5,2)=60 maneiras de escolher
 as frutas que aparecerão no meu resultado.
 ii) Mas ainda temos que determinar a ordem em que as frutas aparecerão
 no resultado. Há 4 lugares para Y, restam 3 lugares para Z e os outros
 têm de ser X. Ou seja, para cada escolha das frutas X, Y e Z que vão
 aparecer (onde X é a letra a ser repetida), há 4.3=12 maneiras de
 posicioná-las.
 iii) Juntando tudo, são 60.12=720 possíveis resultados do tipo um
 par. Como eles são todos igualmente prováveis, a probabilidade pedida
 é 720/1296=5/9.

 Bom, espero não ter errado bobagens, estou meio sem tempo para
 conferir o que escrevi.

 Abraço,
 Ralph

 2009/5/7 Thelio Gama teliog...@gmail.com:
  Bom dia Professores,
  estou bastante confuso com o seguinte problema e agradeço se puderem
 fazer a
  gentileza de explicá-lo :
  Numa máquina de caça-níquel, cada resultado é formado por 4 quaisquer de
 6
  frutas diferentes, podendo haver repetição. Calcule a probabilidade de um
  resultado apresentar duas frutas iguais e 

[obm-l] Re: [obm-l] Re: [obm-l] Re: [obm-l] probabilidades c aça-niquel

2009-05-10 Por tôpico Ralph Teixeira
Eh, aquele enunciado original ganhou nota 4 (de 10) na ENTOR (Escala de
Nariz Torcido do Ralph). Nao eh TERRIVEL, mas poderia ser melhor.

A do Palmerim nao me faz torcer o nariz nao (ENTOR=0). Ainda temos que
pressupor que as frutas sao equiprovaveis (o que me incomoda um pouco, na
barriga, mas o nariz ficou reto) e que as 4 frutas que aparecem sao
independentes entre si (esta nao me incomoda tanto, eh uma hipotese mais
natural). Tambem, se fosse para ser 100% rigoroso, ia ficar um enunciado
muito feio... :) :)

Abraco,
 Ralph

P.S: Ainda to tentando ver se dah para ser mais rigoroso sem perder a
clareza. Consegui:
 Numa máquina de caça-níquel, cada resultado é formado por 4 simbolos
independentes. Cada simbolo eh escolhido ao acaso dentre 6 frutas
diferentes. Calcule a probabilidade de um resultado apresentar duas frutas
iguais e duas outras frutas diferentes entre si.
Acho que ficou mais feio, talvez ateh menos claro... O nariz deu uma fungada
aqui, mas ficou no 0 Mas, pelo menos, o enunciado deixa explicito que os
simbolos sao independentes e as frutas igualmente provaveis (ao acaso
**costuma** indicar isto, o que tambem eh discutivel).
Note que agora nao eh necessario dizer que os simbolos podem repetir -- isto
eh uma consequencia da independencia dos simbolos. :) :)

2009/5/10 Palmerim Soares palmerimsoa...@gmail.com

 Olá Thelio e Mestre Ralph
 Muito pertinentes os comentários do Ralph. Realmente, deve haver extremo
 esmero na formulação dos enunciados, sobretudo quando o assunto é
 probabilidades, que costuma causar confusão justamente por causa das
 ambiguidades nos enunciados.

 Bem, não custa, agora, apresentar uma solução diferente para esta questão:

 Exibir um resultado é uma tarefa que pode ser realizada em 3 etapas
 sucessivas:
 1ª Etapa) Escolha das 3 frutas que irão comparecer no resultado: C(6,3)=20;
 2ª Etapa) Escolha da fruta que comparecerá 2 vezes no resultado: C(3,1)=3;
 3ª Etapa) Escolha das 2 posições destinadas às frutas desiguais: A(4,2)=12
 Pelo Princípio Fundamental da Contagem há: 20 X 3 X 12 = 720 possíveis
 resultados, e, como escreveu o Ralph, já que eles são todos igualmente
 prováveis, a probabilidade pedida
 é 720/1296=5/9.
 Vale notar que a 3ª etapa poderia ser: posicionar as 4 frutas, ou seja,
 fazer uma permutação de 4 elementos, sendo 2 repetidos, o que equivaleria
 exatamente a calcular os anagramas da palavra BALA. (B de Banana, A de
 Amora, L de Laranja e A de Ameixa)
 Então (só para o Thelio ter uma visão geral) o que se quer nesse problema,
 em última análise, é fazer uma permutação de 4 elementos, sendo que dois
 deles são iguais entre si. Mas antes de fazer essa permutação com
 elementos repetidos, precisamos escolher as frutas, o que foi feito nas
 etapas 1 e 2.

 Finalmente, acho que um enunciado que seria aprovado pelo Mestre Ralph
 seria assim:
 (corrija-me se estiver errado, mestre, porque quero aplicar essa questão em
 um simulado)

  Numa máquina de caça-níquel, cada resultado é formado por 4 quaisquer de
 6 frutas diferentes. Supondo que um resultado pode apresentar frutas
 repetidas, calcule a probabilidade de um resultado apresentar duas frutas
 iguais e duas outras frutas diferentes entre si.

 Abraços,
 Palmerim


 2009/5/8 Ralph Teixeira ralp...@gmail.com

 Oi, Thelio.

 Vamos fazer as seguintes hipóteses:

 a) O resultado é formado por 4 símbolos; (isto está bem explícito em
 4 quaisquer...)
 b) Cada símbolo pode ser uma de seis frutas, que designarei por A, B,
 C, D, E, F (também razoavelmente explícito em de 6 frutas
 diferentes...);
 c) Um resultado pode apresentar símbolos iguais (por exemplo, pode ser
 AADE) -- isto está dito, mas com um português ligeiramente ambíguo;
 digo isso pois **gramaticalmente** podendo haver repetição poderia
 se referir a 4 símbolos ou a 6 frutas... mas faz mais sentido se
 for 4 símbolos, podendo haver repetição, que é a minha
 interpretação; a outra interpretação, 6 frutas diferentes, podendo
 haver repetição é meio contraditória...
 d) Em cada símbolo, cada fruta tem a mesma probabilidade de aparecer
 (razoável, mas não é nem um pouco óbvio; aliás, só vou supor isso
 porque tenho que resolver o problema e ele não indicou as
 probabilidades de cada fruta; num caça-níqueis de verdade, isto não
 costuma ser verdadeiro);
 e) Os 4 símbolos são independentes entre si, isto é, o símbolo que
 aparece na primeira janela não afeta de maneira alguma o símbolo da
 segunda (bem razoável, mas também não é certo no caso geral).
 f) O que o enunciado quer é a probabilidade de aparecerem 3 frutas
 distintas, sendo uma delas repetida (se eu quisesse ser muito muito
 chato, diria que AABB tem duas frutas AA iguais e duas frutas BB
 desiguais **da primeira** -- não acho que era isso que o enunciado
 tinha em mente, acho que eles querem dizer, duas frutas iguais e
 duas OUTRAS, desiguais ENTRE SI.). Em linguagem de pôquer: qual é a
 chance de dar um par?

 Agora sim, com tudo destrinchado, eu consigo resolver o 

[obm-l] probabilidades caça-niquel

2009-05-08 Por tôpico Thelio Gama
Bom dia Professores,
estou bastante confuso com o seguinte problema e agradeço se puderem fazer a
gentileza de explicá-lo :

Numa máquina de caça-níquel, cada resultado é formado por 4 quaisquer de 6
frutas diferentes, podendo haver repetição. Calcule a probabilidade de um
resultado apresentar duas frutas iguais e outras duas desiguais.

Obrigado,

Thelio


[obm-l] Re: [obm-l] probabilidades caça-niquel

2009-05-08 Por tôpico Ralph Teixeira
Oi, Thelio.

Vamos fazer as seguintes hipóteses:

a) O resultado é formado por 4 símbolos; (isto está bem explícito em
4 quaisquer...)
b) Cada símbolo pode ser uma de seis frutas, que designarei por A, B,
C, D, E, F (também razoavelmente explícito em de 6 frutas
diferentes...);
c) Um resultado pode apresentar símbolos iguais (por exemplo, pode ser
AADE) -- isto está dito, mas com um português ligeiramente ambíguo;
digo isso pois **gramaticalmente** podendo haver repetição poderia
se referir a 4 símbolos ou a 6 frutas... mas faz mais sentido se
for 4 símbolos, podendo haver repetição, que é a minha
interpretação; a outra interpretação, 6 frutas diferentes, podendo
haver repetição é meio contraditória...
d) Em cada símbolo, cada fruta tem a mesma probabilidade de aparecer
(razoável, mas não é nem um pouco óbvio; aliás, só vou supor isso
porque tenho que resolver o problema e ele não indicou as
probabilidades de cada fruta; num caça-níqueis de verdade, isto não
costuma ser verdadeiro);
e) Os 4 símbolos são independentes entre si, isto é, o símbolo que
aparece na primeira janela não afeta de maneira alguma o símbolo da
segunda (bem razoável, mas também não é certo no caso geral).
f) O que o enunciado quer é a probabilidade de aparecerem 3 frutas
distintas, sendo uma delas repetida (se eu quisesse ser muito muito
chato, diria que AABB tem duas frutas AA iguais e duas frutas BB
desiguais **da primeira** -- não acho que era isso que o enunciado
tinha em mente, acho que eles querem dizer, duas frutas iguais e
duas OUTRAS, desiguais ENTRE SI.). Em linguagem de pôquer: qual é a
chance de dar um par?

Agora sim, com tudo destrinchado, eu consigo resolver o problema. Há
6.6.6.6=1296 possíveis resultados, todos igualmente prováveis graças a
(d) e (e). Quantos são da forma XXYZ (ou permutações)?
i) Primeiro, vou escolher as frutas que vão aparecer na minha
sequencia: note que X é bem distinto de Y e Z, que são intercambiáveis
neste momento. Há 6 maneiras de escolher X; agora, há C(5,2) maneiras
de escolher as frutas Y e Z. Então há 6.C(5,2)=60 maneiras de escolher
as frutas que aparecerão no meu resultado.
ii) Mas ainda temos que determinar a ordem em que as frutas aparecerão
no resultado. Há 4 lugares para Y, restam 3 lugares para Z e os outros
têm de ser X. Ou seja, para cada escolha das frutas X, Y e Z que vão
aparecer (onde X é a letra a ser repetida), há 4.3=12 maneiras de
posicioná-las.
iii) Juntando tudo, são 60.12=720 possíveis resultados do tipo um
par. Como eles são todos igualmente prováveis, a probabilidade pedida
é 720/1296=5/9.

Bom, espero não ter errado bobagens, estou meio sem tempo para
conferir o que escrevi.

Abraço,
 Ralph

2009/5/7 Thelio Gama teliog...@gmail.com:
 Bom dia Professores,
 estou bastante confuso com o seguinte problema e agradeço se puderem fazer a
 gentileza de explicá-lo :
 Numa máquina de caça-níquel, cada resultado é formado por 4 quaisquer de 6
 frutas diferentes, podendo haver repetição. Calcule a probabilidade de um
 resultado apresentar duas frutas iguais e outras duas desiguais.
 Obrigado,
 Thelio

=
Instruções para entrar na lista, sair da lista e usar a lista em
http://www.mat.puc-rio.br/~obmlistas/obm-l.html
=


[obm-l] Re: [obm-l] Probabilidades Geométricas: 2 proble mas difíceis

2008-07-16 Por tôpico Chicao Valadares
estou reenviando pq acho que eu enviei e nao chegou


--- Em sex, 11/7/08, Chicao Valadares [EMAIL PROTECTED] escreveu:

 De: Chicao Valadares [EMAIL PROTECTED]
 Assunto: Re: [obm-l] Re: [obm-l] Re: [obm-l] Re: [obm-l] Probabilidades 
 Geométricas: 2 problemas difíceis
 Para: obm-l@mat.puc-rio.br
 Data: Sexta-feira, 11 de Julho de 2008, 12:07
 vou postar a minha solução:
 
 Vc sorteia de maneira uniforme e independente dois pontos x
 e y no segmento [0,1], obtendo, as três únicas
 possibilidades seguintes:
 
 (I) x = y com probabilidade de 1/3;
 (II) x  y com probabilidade de 1/3;
 (III) x  y com probabilidade de 1/3;
 
 Vamos trabalhar o III:
 
 Obteremos então os subsegmentos x, y-x e 1-y.
 Para que esses subsegmentos formem lados de um triangulo é
 condição necessária e suficiente que as seguintes três
 condições ocorram:
 (a) x + y-x  1-y donde y  1/2;
 (b) x + 1-y  y-x donde y - x  1/2;
 (c) y-x + 1-y  x donde x  1/2;
 
 Como trata-se do intervalo [0, 1] e o sorteio é de maneira
 uniforme e independente não é difícil ver que a
 probabilidade tanto de a, como de b e
 de c é 1/2.  
 
 Daí como o  sorteio é de maneira uniforme e independente,
 III mais a,b e c ocorrem com a seguinte probabilidade :
 1/3 vezes 1/2 vezes 1/2 vezes 1/2 = 1/24
 
 Analogamente para que II ocorra e seus subsegmentos formem
 um triangulo deve ocorrer com probabilidade igual a 1/24.
 
 Como I não forma triângulo então deveremos apenas
 contabilizar II e III então a probabilidade será 1/24 +
 1/24 = 1/12 !!! 
 
 Ou eu errei ou vocês erraram ou nós erramos, peço para
 verificarem a minha solução, eu acho que vocês não
 levaram em consideração a probabilidade de 
 x = y. 
 
 
 
 O Binômio de Newton é tão belo como a Vênus de
 Milo.
 O que há é pouca gente para dar por isso... 
 Fernando Pessoa - Poesias de Alvaro Campos
 
 _
 As informações existentes nessa mensagem e no(s)
 arquivo(s) anexado(s) 
 são
 para uso restrito, sendo seu sigilo protegido por lei. Caso
 não seja
 destinatário, saiba que leitura, divulgação ou cópia
 são proibidas. 
 Favor
 apagar as informações e notificar o remetente. O uso
 impróprio será 
 tratado
 conforme as normas da empresa e a legislação em vigor.
 Agradecemos sua
 colaboração.
 
 
 The information mentioned in this message and in the
 archives attached 
 are
 of restricted use, and its privacy is protected by law. If
 you are not 
 the
 addressee, be aware that reading, disclosure or copy are
 forbidden. 
 Please
 delete this information and notify the sender.
 Inappropriate use will 
 be
 tracted according to company's rules and valid laws.
 Thank you for your
 cooperation.
 
 
 --- Em qui, 10/7/08, Rogerio Ponce
 [EMAIL PROTECTED] escreveu:
 
  De: Rogerio Ponce [EMAIL PROTECTED]
  Assunto: Re: [obm-l] Re: [obm-l] Re: [obm-l] Re:
 [obm-l] Probabilidades Geométricas: 2 problemas difíceis
  Para: obm-l@mat.puc-rio.br
  Data: Quinta-feira, 10 de Julho de 2008, 18:34
  E' verdade Ralph,
  nossas solucoes sao praticamente a mesma coisa, mas a
 sua
  esta'
  muuuito mais artistica que a minha...:)
  Abracao,
  Rogerio Ponce
  
  PS: e' por essas e outras que tenho certeza de que
 voce
  vai gostar de
  resolver o Barango...
  
  
  
  
  
  2008/7/10 Ralph Teixeira [EMAIL PROTECTED]:
   Este problema eh legal, e jah apareceu um par de
 vezes
  na lista. A minha
   solucao eh igualzinha aa do Ponce, mas a
 **MII-NHA**
  tem uma figuri-inha, a
   do Pon-ce **NAO TE-EM!!**. :P
   Aqui estah ela, para que todos apreciem meus
 dotes
  artisticos:
  
 
 http://www.mat.puc-rio.br/~nicolau/olimp/obm-l.200706/msg00182.html
  
   Abraco, Ralph.
  
   P.S.: Eh, por causa destes dotes artisticos eh
 que eu
  fui fazer
   Matematica :)
  
   2008/7/10 Chicao Valadares
  [EMAIL PROTECTED]:
   Eu fiz algo parecido e achei 1/12. Depois eu
 posto
  aqui na lista.
  
  
   O Binômio de Newton é tão belo como
 a
  Vênus de Milo.
   O que há é pouca gente para dar por isso...
  
   Fernando Pessoa - Poesias de Alvaro Campos
  
  
 
 _
  
   --- Em seg, 7/7/08, Rogerio Ponce
  [EMAIL PROTECTED] escreveu:
  
   De: Rogerio Ponce
 [EMAIL PROTECTED]
   Assunto: Re: [obm-l] Re: [obm-l] Re:
 [obm-l]
  Probabilidades Geométricas:
   2 problemas difíceis
   Para: obm-l@mat.puc-rio.br
   Data: Segunda-feira, 7 de Julho de 2008,
 20:38
   Ola' Chicao,
   sem perda de generalidade, eu assumi que
 o
  segmento
   de reta do
   problema seria o segmento unitario [0 1],
 de
  forma que
   x pode ser
   qualquer real no intervalo [0, 1].
   E para cada valor de x, o
 ponto
  y
   tambem pode estar em qualquer
   posicao no intervalo [0, 1].
   Assim, usando o espaco cartesiano para
 plotar
  todos os
   pares (x,y)
   possiveis, voce obtera' um quadrado
 de
  lado unitario.
   Da mesma forma, se voce plotar todos os
 pares
  que
   satisfazem 'as
   exigencias do problema, voce obtera' 
 os
  dois

Re: [obm-l] Re: [obm-l] Probabilidades Geométricas: 2 problemas difíceis

2008-07-16 Por tôpico Rogerio Ponce
Ola' Chicao,
reveja as 3 mensagens que mandei em resposta 'a sua solucao:

http://www.mail-archive.com/obm-l@mat.puc-rio.br/msg42361.html

http://www.mail-archive.com/obm-l@mat.puc-rio.br/msg42362.html

http://www.mail-archive.com/obm-l@mat.puc-rio.br/msg42374.html


[]'s
Rogerio Ponce


2008/7/16 Chicao Valadares [EMAIL PROTECTED]:
 estou reenviando pq acho que eu enviei e nao chegou


 --- Em sex, 11/7/08, Chicao Valadares [EMAIL PROTECTED] escreveu:

=
Instruções para entrar na lista, sair da lista e usar a lista em
http://www.mat.puc-rio.br/~obmlistas/obm-l.html
=


Re: [obm-l] Re: [obm-l] Re: [obm-l] Re: [obm-l] Probabilidades Geométricas: 2 problemas difíceis

2008-07-13 Por tôpico Rogerio Ponce
Oi Chicao,
o programinha abaixo serve para dar uma ideia aproximada do resultado correto.

Ele simula 10 sorteios de x,y , e imprime a razao entre o numero
de triangulos obtidos e o total de experimentos.

Para ser compilado em Linux (ou outro Unix) utilize gcc prog.c -lm.
Para ser compilado em algum outro SO, provavelmente voce precisara'
acrescentar/alterar alguma linha no codigo, mas sera' tudo muito
simples.

[]'s
Rogerio Ponce

=== prog.c =

#include stdio.h
#include stdlib.h
#define TOTAL_EXPERIMENTOS 10
main()
{
int i,count_ok;
float x,y,a,b,c;

/* Inicializa o gerador de numeros pseudorandomicos com um
inteiro qualquer */
srand48( (long int) 65269);

/* Executa os experimentos */
for(count_ok=0,i=0;iTOTAL_EXPERIMENTOS;i++){

/* Faz o sorteio de 2 pontos em [0,1] */
x = (float)drand48();
y = (float)drand48();

/* Calcula os 3 segmentos a,b,c definidos pelo sorteio */
if(xy) {
a=x;
b=y-x;
c=1.-y;
} else {
a=y;
b=x-y;
c=1.-x;
}

/* Testa se a,b,c definem um triangulo. Caso
afirmativo incrementa o contador */
if( (ab+c)  (ba+c)  (ca+b) ) count_ok++;
}

/* Imprime a relacao entre os experimentos com sucesso e o
total de experimentos */
fprintf(stdout,Relacao = %.4f\n, count_ok/(float)TOTAL_EXPERIMENTOS );
}

==


Em 11/07/08, Rogerio Ponce[EMAIL PROTECTED] escreveu:
 Ola' Chicao,
 na mesma solucao, voce ainda se engana ao considerar que as condicoes
 a, b  e c  sejam independentes entre si, com probabilidade 1/2
 cada uma.
 Acontece que elas nao sao independentes!
 Exemplo: voce nao consegue ter, simultaneamente, as condicoes a e b
 falsas.
 []'s
 Rogerio Ponce.




 2008/7/11 Rogerio Ponce [EMAIL PROTECTED]:
 Oi Chicao,
 o caso I tem probabilidade ZERO.
 So' pra deixar sua intuicao trabalhar, imagine que a maneira
 uniforme de obter um ponto no intervalo [0,1] signifique obter um
 numero real com 6 casas decimais neste intervalo. Portanto, existe um
 milhao de resultados diferentes para um sorteio. Sera' que a
 possibilidade de se obter duas vezes o mesmo valor e' 1/3?
 Agora imagine que em vez de apenas um milhao, isso tenda para infinito...

 []'s
 Rogerio Ponce



 2008/7/11 Chicao Valadares [EMAIL PROTECTED]:
 vou postar a minha solução:

 Vc sorteia de maneira uniforme e independente dois pontos x e y no
 segmento [0,1], obtendo, as três únicas possibilidades seguintes:

 (I) x = y com probabilidade de 1/3;
 (II) x  y com probabilidade de 1/3;
 (III) x  y com probabilidade de 1/3;

 Vamos trabalhar o III:

 Obteremos então os subsegmentos x, y-x e 1-y.
 Para que esses subsegmentos formem lados de um triangulo é condição
 necessária e suficiente que as seguintes três condições ocorram:
 (a) x + y-x  1-y donde y  1/2;
 (b) x + 1-y  y-x donde y - x  1/2;
 (c) y-x + 1-y  x donde x  1/2;

 Como trata-se do intervalo [0, 1] e o sorteio é de maneira uniforme e
 independente não é difícil ver que a probabilidade tanto de a, como de b
 e
 de c é 1/2.

 Daí como o  sorteio é de maneira uniforme e independente, III mais a,b e
 c ocorrem com a seguinte probabilidade :
 1/3 vezes 1/2 vezes 1/2 vezes 1/2 = 1/24

 Analogamente para que II ocorra e seus subsegmentos formem um triangulo
 deve ocorrer com probabilidade igual a 1/24.

 Como I não forma triângulo então deveremos apenas contabilizar II e III
 então a probabilidade será 1/24 + 1/24 = 1/12 !!!

 Ou eu errei ou vocês erraram ou nós erramos, peço para verificarem a
 minha solução, eu acho que vocês não levaram em consideração a
 probabilidade de
 x = y.



=
Instruções para entrar na lista, sair da lista e usar a lista em
http://www.mat.puc-rio.br/~obmlistas/obm-l.html
=


[obm-l] Re: [obm-l] Re: [obm-l] Re: [obm-l] Re : [obm-l] Probabilidades Geométricas: 2 problema s difíceis

2008-07-11 Por tôpico Chicao Valadares
vou postar a minha solução:

Vc sorteia de maneira uniforme e independente dois pontos x e y no segmento 
[0,1], obtendo, as três únicas possibilidades seguintes:

(I) x = y com probabilidade de 1/3;
(II) x  y com probabilidade de 1/3;
(III) x  y com probabilidade de 1/3;

Vamos trabalhar o III:

Obteremos então os subsegmentos x, y-x e 1-y.
Para que esses subsegmentos formem lados de um triangulo é condição necessária 
e suficiente que as seguintes três condições ocorram:
(a) x + y-x  1-y donde y  1/2;
(b) x + 1-y  y-x donde y - x  1/2;
(c) y-x + 1-y  x donde x  1/2;

Como trata-se do intervalo [0, 1] e o sorteio é de maneira uniforme e 
independente não é difícil ver que a probabilidade tanto de a, como de b e
de c é 1/2.  

Daí como o  sorteio é de maneira uniforme e independente, III mais a,b e c 
ocorrem com a seguinte probabilidade :
1/3 vezes 1/2 vezes 1/2 vezes 1/2 = 1/24

Analogamente para que II ocorra e seus subsegmentos formem um triangulo deve 
ocorrer com probabilidade igual a 1/24.

Como I não forma triângulo então deveremos apenas contabilizar II e III então a 
probabilidade será 1/24 + 1/24 = 1/12 !!! 

Ou eu errei ou vocês erraram ou nós erramos, peço para verificarem a minha 
solução, eu acho que vocês não levaram em consideração a probabilidade de 
x = y. 



O Binômio de Newton é tão belo como a Vênus de Milo.
O que há é pouca gente para dar por isso... 
Fernando Pessoa - Poesias de Alvaro Campos

_
As informações existentes nessa mensagem e no(s) arquivo(s) anexado(s) 
são
para uso restrito, sendo seu sigilo protegido por lei. Caso não seja
destinatário, saiba que leitura, divulgação ou cópia são proibidas. 
Favor
apagar as informações e notificar o remetente. O uso impróprio será 
tratado
conforme as normas da empresa e a legislação em vigor. Agradecemos sua
colaboração.


The information mentioned in this message and in the archives attached 
are
of restricted use, and its privacy is protected by law. If you are not 
the
addressee, be aware that reading, disclosure or copy are forbidden. 
Please
delete this information and notify the sender. Inappropriate use will 
be
tracted according to company's rules and valid laws. Thank you for your
cooperation.


--- Em qui, 10/7/08, Rogerio Ponce [EMAIL PROTECTED] escreveu:

 De: Rogerio Ponce [EMAIL PROTECTED]
 Assunto: Re: [obm-l] Re: [obm-l] Re: [obm-l] Re: [obm-l] Probabilidades 
 Geométricas: 2 problemas difíceis
 Para: obm-l@mat.puc-rio.br
 Data: Quinta-feira, 10 de Julho de 2008, 18:34
 E' verdade Ralph,
 nossas solucoes sao praticamente a mesma coisa, mas a sua
 esta'
 muuuito mais artistica que a minha...:)
 Abracao,
 Rogerio Ponce
 
 PS: e' por essas e outras que tenho certeza de que voce
 vai gostar de
 resolver o Barango...
 
 
 
 
 
 2008/7/10 Ralph Teixeira [EMAIL PROTECTED]:
  Este problema eh legal, e jah apareceu um par de vezes
 na lista. A minha
  solucao eh igualzinha aa do Ponce, mas a **MII-NHA**
 tem uma figuri-inha, a
  do Pon-ce **NAO TE-EM!!**. :P
  Aqui estah ela, para que todos apreciem meus dotes
 artisticos:
 
 http://www.mat.puc-rio.br/~nicolau/olimp/obm-l.200706/msg00182.html
 
  Abraco, Ralph.
 
  P.S.: Eh, por causa destes dotes artisticos eh que eu
 fui fazer
  Matematica :)
 
  2008/7/10 Chicao Valadares
 [EMAIL PROTECTED]:
  Eu fiz algo parecido e achei 1/12. Depois eu posto
 aqui na lista.
 
 
  O Binômio de Newton é tão belo como a
 Vênus de Milo.
  O que há é pouca gente para dar por isso...
 
  Fernando Pessoa - Poesias de Alvaro Campos
 
 
 _
 
  --- Em seg, 7/7/08, Rogerio Ponce
 [EMAIL PROTECTED] escreveu:
 
  De: Rogerio Ponce [EMAIL PROTECTED]
  Assunto: Re: [obm-l] Re: [obm-l] Re: [obm-l]
 Probabilidades Geométricas:
  2 problemas difíceis
  Para: obm-l@mat.puc-rio.br
  Data: Segunda-feira, 7 de Julho de 2008, 20:38
  Ola' Chicao,
  sem perda de generalidade, eu assumi que o
 segmento
  de reta do
  problema seria o segmento unitario [0 1], de
 forma que
  x pode ser
  qualquer real no intervalo [0, 1].
  E para cada valor de x, o ponto
 y
  tambem pode estar em qualquer
  posicao no intervalo [0, 1].
  Assim, usando o espaco cartesiano para plotar
 todos os
  pares (x,y)
  possiveis, voce obtera' um quadrado de
 lado unitario.
  Da mesma forma, se voce plotar todos os pares
 que
  satisfazem 'as
  exigencias do problema, voce obtera'  os
 dois
  triangulos internos ao
  quadrado unitario, conforme descrito na
 solucao.
 
  Repare que os tais dois triangulos
 sao
  simplesmente o conjunto de
  pares (x,y) capazes de definir um triangulo
 sobre o
  segmento unitario,
  conforme o enunciado.
  Para isso, e' necessario e suficiente que
 x
  e y satisfacam 'as
  seguintes condicoes:
  - o menor deles e' menor (ou igual**) que
 1/2
  - o maior deles e' maior (ou igual**) que
 1/2
  - a diferenca entre eles e' menor (ou
 igual**) que 1/2
 
  ** OBS: quando acontece um

Re: [obm-l] Re: [obm-l] Re: [obm-l] Re: [obm-l] Re: [obm-l] Probabilidades Geométricas: 2 problemas difíceis

2008-07-11 Por tôpico Rogerio Ponce
Oi Chicao,
o caso I tem probabilidade ZERO.
So' pra deixar sua intuicao trabalhar, imagine que a maneira
uniforme de obter um ponto no intervalo [0,1] signifique obter um
numero real com 6 casas decimais neste intervalo. Portanto, existe um
milhao de resultados diferentes para um sorteio. Sera' que a
possibilidade de se obter duas vezes o mesmo valor e' 1/3?
Agora imagine que em vez de apenas um milhao, isso tenda para infinito...

[]'s
Rogerio Ponce



2008/7/11 Chicao Valadares [EMAIL PROTECTED]:
 vou postar a minha solução:

 Vc sorteia de maneira uniforme e independente dois pontos x e y no segmento 
 [0,1], obtendo, as três únicas possibilidades seguintes:

 (I) x = y com probabilidade de 1/3;
 (II) x  y com probabilidade de 1/3;
 (III) x  y com probabilidade de 1/3;

 Vamos trabalhar o III:

 Obteremos então os subsegmentos x, y-x e 1-y.
 Para que esses subsegmentos formem lados de um triangulo é condição 
 necessária e suficiente que as seguintes três condições ocorram:
 (a) x + y-x  1-y donde y  1/2;
 (b) x + 1-y  y-x donde y - x  1/2;
 (c) y-x + 1-y  x donde x  1/2;

 Como trata-se do intervalo [0, 1] e o sorteio é de maneira uniforme e 
 independente não é difícil ver que a probabilidade tanto de a, como de b e
 de c é 1/2.

 Daí como o  sorteio é de maneira uniforme e independente, III mais a,b e c 
 ocorrem com a seguinte probabilidade :
 1/3 vezes 1/2 vezes 1/2 vezes 1/2 = 1/24

 Analogamente para que II ocorra e seus subsegmentos formem um triangulo deve 
 ocorrer com probabilidade igual a 1/24.

 Como I não forma triângulo então deveremos apenas contabilizar II e III então 
 a probabilidade será 1/24 + 1/24 = 1/12 !!!

 Ou eu errei ou vocês erraram ou nós erramos, peço para verificarem a minha 
 solução, eu acho que vocês não levaram em consideração a probabilidade de
 x = y.



 O Binômio de Newton é tão belo como a Vênus de Milo.
 O que há é pouca gente para dar por isso... 
 Fernando Pessoa - Poesias de Alvaro Campos

 _
 As informações existentes nessa mensagem e no(s) arquivo(s) anexado(s)
 são
 para uso restrito, sendo seu sigilo protegido por lei. Caso não seja
 destinatário, saiba que leitura, divulgação ou cópia são proibidas.
 Favor
 apagar as informações e notificar o remetente. O uso impróprio será
 tratado
 conforme as normas da empresa e a legislação em vigor. Agradecemos sua
 colaboração.


 The information mentioned in this message and in the archives attached
 are
 of restricted use, and its privacy is protected by law. If you are not
 the
 addressee, be aware that reading, disclosure or copy are forbidden.
 Please
 delete this information and notify the sender. Inappropriate use will
 be
 tracted according to company's rules and valid laws. Thank you for your
 cooperation.


 --- Em qui, 10/7/08, Rogerio Ponce [EMAIL PROTECTED] escreveu:

 De: Rogerio Ponce [EMAIL PROTECTED]
 Assunto: Re: [obm-l] Re: [obm-l] Re: [obm-l] Re: [obm-l] Probabilidades 
 Geométricas: 2 problemas difíceis
 Para: obm-l@mat.puc-rio.br
 Data: Quinta-feira, 10 de Julho de 2008, 18:34
 E' verdade Ralph,
 nossas solucoes sao praticamente a mesma coisa, mas a sua
 esta'
 muuuito mais artistica que a minha...:)
 Abracao,
 Rogerio Ponce

 PS: e' por essas e outras que tenho certeza de que voce
 vai gostar de
 resolver o Barango...





 2008/7/10 Ralph Teixeira [EMAIL PROTECTED]:
  Este problema eh legal, e jah apareceu um par de vezes
 na lista. A minha
  solucao eh igualzinha aa do Ponce, mas a **MII-NHA**
 tem uma figuri-inha, a
  do Pon-ce **NAO TE-EM!!**. :P
  Aqui estah ela, para que todos apreciem meus dotes
 artisticos:
 
 http://www.mat.puc-rio.br/~nicolau/olimp/obm-l.200706/msg00182.html
 
  Abraco, Ralph.
 
  P.S.: Eh, por causa destes dotes artisticos eh que eu
 fui fazer
  Matematica :)
 
  2008/7/10 Chicao Valadares
 [EMAIL PROTECTED]:
  Eu fiz algo parecido e achei 1/12. Depois eu posto
 aqui na lista.
 
 
  O Binômio de Newton é tão belo como a
 Vênus de Milo.
  O que há é pouca gente para dar por isso...
 
  Fernando Pessoa - Poesias de Alvaro Campos
 
 
 _
 
  --- Em seg, 7/7/08, Rogerio Ponce
 [EMAIL PROTECTED] escreveu:
 
  De: Rogerio Ponce [EMAIL PROTECTED]
  Assunto: Re: [obm-l] Re: [obm-l] Re: [obm-l]
 Probabilidades Geométricas:
  2 problemas difíceis
  Para: obm-l@mat.puc-rio.br
  Data: Segunda-feira, 7 de Julho de 2008, 20:38
  Ola' Chicao,
  sem perda de generalidade, eu assumi que o
 segmento
  de reta do
  problema seria o segmento unitario [0 1], de
 forma que
  x pode ser
  qualquer real no intervalo [0, 1].
  E para cada valor de x, o ponto
 y
  tambem pode estar em qualquer
  posicao no intervalo [0, 1].
  Assim, usando o espaco cartesiano para plotar
 todos os
  pares (x,y)
  possiveis, voce obtera' um quadrado de
 lado unitario.
  Da mesma forma, se voce plotar todos os pares
 que
  satisfazem 'as
  exigencias do problema, voce obtera'  os

Re: [obm-l] Re: [obm-l] Re: [obm-l] Re: [obm-l] Re: [obm-l] Probabilidades Geométricas: 2 problemas difíceis

2008-07-11 Por tôpico Rogerio Ponce
Ola' Chicao,
na mesma solucao, voce ainda se engana ao considerar que as condicoes
a, b  e c  sejam independentes entre si, com probabilidade 1/2
cada uma.
Acontece que elas nao sao independentes!
Exemplo: voce nao consegue ter, simultaneamente, as condicoes a e b falsas.
[]'s
Rogerio Ponce.




2008/7/11 Rogerio Ponce [EMAIL PROTECTED]:
 Oi Chicao,
 o caso I tem probabilidade ZERO.
 So' pra deixar sua intuicao trabalhar, imagine que a maneira
 uniforme de obter um ponto no intervalo [0,1] signifique obter um
 numero real com 6 casas decimais neste intervalo. Portanto, existe um
 milhao de resultados diferentes para um sorteio. Sera' que a
 possibilidade de se obter duas vezes o mesmo valor e' 1/3?
 Agora imagine que em vez de apenas um milhao, isso tenda para infinito...

 []'s
 Rogerio Ponce



 2008/7/11 Chicao Valadares [EMAIL PROTECTED]:
 vou postar a minha solução:

 Vc sorteia de maneira uniforme e independente dois pontos x e y no segmento 
 [0,1], obtendo, as três únicas possibilidades seguintes:

 (I) x = y com probabilidade de 1/3;
 (II) x  y com probabilidade de 1/3;
 (III) x  y com probabilidade de 1/3;

 Vamos trabalhar o III:

 Obteremos então os subsegmentos x, y-x e 1-y.
 Para que esses subsegmentos formem lados de um triangulo é condição 
 necessária e suficiente que as seguintes três condições ocorram:
 (a) x + y-x  1-y donde y  1/2;
 (b) x + 1-y  y-x donde y - x  1/2;
 (c) y-x + 1-y  x donde x  1/2;

 Como trata-se do intervalo [0, 1] e o sorteio é de maneira uniforme e 
 independente não é difícil ver que a probabilidade tanto de a, como de b e
 de c é 1/2.

 Daí como o  sorteio é de maneira uniforme e independente, III mais a,b e c 
 ocorrem com a seguinte probabilidade :
 1/3 vezes 1/2 vezes 1/2 vezes 1/2 = 1/24

 Analogamente para que II ocorra e seus subsegmentos formem um triangulo deve 
 ocorrer com probabilidade igual a 1/24.

 Como I não forma triângulo então deveremos apenas contabilizar II e III 
 então a probabilidade será 1/24 + 1/24 = 1/12 !!!

 Ou eu errei ou vocês erraram ou nós erramos, peço para verificarem a minha 
 solução, eu acho que vocês não levaram em consideração a probabilidade de
 x = y.



 O Binômio de Newton é tão belo como a Vênus de Milo.
 O que há é pouca gente para dar por isso... 
 Fernando Pessoa - Poesias de Alvaro Campos

 _
 As informações existentes nessa mensagem e no(s) arquivo(s) anexado(s)
 são
 para uso restrito, sendo seu sigilo protegido por lei. Caso não seja
 destinatário, saiba que leitura, divulgação ou cópia são proibidas.
 Favor
 apagar as informações e notificar o remetente. O uso impróprio será
 tratado
 conforme as normas da empresa e a legislação em vigor. Agradecemos sua
 colaboração.


 The information mentioned in this message and in the archives attached
 are
 of restricted use, and its privacy is protected by law. If you are not
 the
 addressee, be aware that reading, disclosure or copy are forbidden.
 Please
 delete this information and notify the sender. Inappropriate use will
 be
 tracted according to company's rules and valid laws. Thank you for your
 cooperation.


 --- Em qui, 10/7/08, Rogerio Ponce [EMAIL PROTECTED] escreveu:

 De: Rogerio Ponce [EMAIL PROTECTED]
 Assunto: Re: [obm-l] Re: [obm-l] Re: [obm-l] Re: [obm-l] Probabilidades 
 Geométricas: 2 problemas difíceis
 Para: obm-l@mat.puc-rio.br
 Data: Quinta-feira, 10 de Julho de 2008, 18:34
 E' verdade Ralph,
 nossas solucoes sao praticamente a mesma coisa, mas a sua
 esta'
 muuuito mais artistica que a minha...:)
 Abracao,
 Rogerio Ponce

 PS: e' por essas e outras que tenho certeza de que voce
 vai gostar de
 resolver o Barango...





 2008/7/10 Ralph Teixeira [EMAIL PROTECTED]:
  Este problema eh legal, e jah apareceu um par de vezes
 na lista. A minha
  solucao eh igualzinha aa do Ponce, mas a **MII-NHA**
 tem uma figuri-inha, a
  do Pon-ce **NAO TE-EM!!**. :P
  Aqui estah ela, para que todos apreciem meus dotes
 artisticos:
 
 http://www.mat.puc-rio.br/~nicolau/olimp/obm-l.200706/msg00182.html
 
  Abraco, Ralph.
 
  P.S.: Eh, por causa destes dotes artisticos eh que eu
 fui fazer
  Matematica :)
 
  2008/7/10 Chicao Valadares
 [EMAIL PROTECTED]:
  Eu fiz algo parecido e achei 1/12. Depois eu posto
 aqui na lista.
 
 
  O Binômio de Newton é tão belo como a
 Vênus de Milo.
  O que há é pouca gente para dar por isso...
 
  Fernando Pessoa - Poesias de Alvaro Campos
 
 
 _
 
  --- Em seg, 7/7/08, Rogerio Ponce
 [EMAIL PROTECTED] escreveu:
 
  De: Rogerio Ponce [EMAIL PROTECTED]
  Assunto: Re: [obm-l] Re: [obm-l] Re: [obm-l]
 Probabilidades Geométricas:
  2 problemas difíceis
  Para: obm-l@mat.puc-rio.br
  Data: Segunda-feira, 7 de Julho de 2008, 20:38
  Ola' Chicao,
  sem perda de generalidade, eu assumi que o
 segmento
  de reta do
  problema seria o segmento unitario [0 1], de
 forma que
  x pode ser
  qualquer real no intervalo

Re: [obm-l] Re: [obm-l] Re: [obm-l] Re: [obm-l] Probabilidades Geométricas: 2 problemas difíceis

2008-07-10 Por tôpico Ralph Teixeira
Este problema eh legal, e jah apareceu um par de vezes na lista. A minha
solucao eh igualzinha aa do Ponce, mas a **MII-NHA** tem uma figuri-inha, a
do Pon-ce **NAO TE-EM!!**. :P
Aqui estah ela, para que todos apreciem meus dotes artisticos:
http://www.mat.puc-rio.br/~nicolau/olimp/obm-l.200706/msg00182.html

 Abraco, Ralph.

P.S.: Eh, por causa destes dotes artisticos eh que eu fui fazer
Matematica :)

2008/7/10 Chicao Valadares [EMAIL PROTECTED]:
 Eu fiz algo parecido e achei 1/12. Depois eu posto aqui na lista.


 O Binômio de Newton é tão belo como a Vênus de Milo.
 O que há é pouca gente para dar por isso... 
 Fernando Pessoa - Poesias de Alvaro Campos

 _
 As informações existentes nessa mensagem e no(s) arquivo(s) anexado(s)
 são
 para uso restrito, sendo seu sigilo protegido por lei. Caso não seja
 destinatário, saiba que leitura, divulgação ou cópia são proibidas.
 Favor
 apagar as informações e notificar o remetente. O uso impróprio será
 tratado
 conforme as normas da empresa e a legislação em vigor. Agradecemos sua
 colaboração.


 The information mentioned in this message and in the archives attached
 are
 of restricted use, and its privacy is protected by law. If you are not
 the
 addressee, be aware that reading, disclosure or copy are forbidden.
 Please
 delete this information and notify the sender. Inappropriate use will
 be
 tracted according to company's rules and valid laws. Thank you for your
 cooperation.


 --- Em seg, 7/7/08, Rogerio Ponce [EMAIL PROTECTED] escreveu:

 De: Rogerio Ponce [EMAIL PROTECTED]
 Assunto: Re: [obm-l] Re: [obm-l] Re: [obm-l] Probabilidades Geométricas:
2 problemas difíceis
 Para: obm-l@mat.puc-rio.br
 Data: Segunda-feira, 7 de Julho de 2008, 20:38
 Ola' Chicao,
 sem perda de generalidade, eu assumi que o segmento
 de reta do
 problema seria o segmento unitario [0 1], de forma que
 x pode ser
 qualquer real no intervalo [0, 1].
 E para cada valor de x, o ponto y
 tambem pode estar em qualquer
 posicao no intervalo [0, 1].
 Assim, usando o espaco cartesiano para plotar todos os
 pares (x,y)
 possiveis, voce obtera' um quadrado de lado unitario.
 Da mesma forma, se voce plotar todos os pares que
 satisfazem 'as
 exigencias do problema, voce obtera'  os dois
 triangulos internos ao
 quadrado unitario, conforme descrito na solucao.

 Repare que os tais dois triangulos sao
 simplesmente o conjunto de
 pares (x,y) capazes de definir um triangulo sobre o
 segmento unitario,
 conforme o enunciado.
 Para isso, e' necessario e suficiente que x
 e y satisfacam 'as
 seguintes condicoes:
 - o menor deles e' menor (ou igual**) que 1/2
 - o maior deles e' maior (ou igual**) que 1/2
 - a diferenca entre eles e' menor (ou igual**) que 1/2

 ** OBS: quando acontece um igual , temos um
 triangulo degenerado
 (com area zero).

 []'s
 Rogerio Ponce.



 2008/7/7 Chicao Valadares
 [EMAIL PROTECTED]:
  Os valores possiveis de x e y equivalem a area
 do quadrado unitario,
   que vale 1.
 
  Nao entendi, seria o produto xy que equivaleria a
 área?
 
 

 =
 Instruções para entrar na lista, sair da lista e usar a
 lista em
 http://www.mat.puc-rio.br/~obmlistas/obm-l.html
 =


  Novos endereços, o Yahoo! que você conhece. Crie um email novo com a
sua cara @ymail.com ou @rocketmail.com.
 http://br.new.mail.yahoo.com/addresses

 =
 Instruções para entrar na lista, sair da lista e usar a lista em
 http://www.mat.puc-rio.br/~obmlistas/obm-l.html
 =



Re: [obm-l] Re: [obm-l] Re: [obm-l] Re: [obm-l] Probabilidades Geométricas: 2 problemas difíceis

2008-07-10 Por tôpico Rogerio Ponce
E' verdade Ralph,
nossas solucoes sao praticamente a mesma coisa, mas a sua esta'
muuuito mais artistica que a minha...:)
Abracao,
Rogerio Ponce

PS: e' por essas e outras que tenho certeza de que voce vai gostar de
resolver o Barango...





2008/7/10 Ralph Teixeira [EMAIL PROTECTED]:
 Este problema eh legal, e jah apareceu um par de vezes na lista. A minha
 solucao eh igualzinha aa do Ponce, mas a **MII-NHA** tem uma figuri-inha, a
 do Pon-ce **NAO TE-EM!!**. :P
 Aqui estah ela, para que todos apreciem meus dotes artisticos:
 http://www.mat.puc-rio.br/~nicolau/olimp/obm-l.200706/msg00182.html

 Abraco, Ralph.

 P.S.: Eh, por causa destes dotes artisticos eh que eu fui fazer
 Matematica :)

 2008/7/10 Chicao Valadares [EMAIL PROTECTED]:
 Eu fiz algo parecido e achei 1/12. Depois eu posto aqui na lista.


 O Binômio de Newton é tão belo como a Vênus de Milo.
 O que há é pouca gente para dar por isso... 
 Fernando Pessoa - Poesias de Alvaro Campos

 _

 --- Em seg, 7/7/08, Rogerio Ponce [EMAIL PROTECTED] escreveu:

 De: Rogerio Ponce [EMAIL PROTECTED]
 Assunto: Re: [obm-l] Re: [obm-l] Re: [obm-l] Probabilidades Geométricas:
 2 problemas difíceis
 Para: obm-l@mat.puc-rio.br
 Data: Segunda-feira, 7 de Julho de 2008, 20:38
 Ola' Chicao,
 sem perda de generalidade, eu assumi que o segmento
 de reta do
 problema seria o segmento unitario [0 1], de forma que
 x pode ser
 qualquer real no intervalo [0, 1].
 E para cada valor de x, o ponto y
 tambem pode estar em qualquer
 posicao no intervalo [0, 1].
 Assim, usando o espaco cartesiano para plotar todos os
 pares (x,y)
 possiveis, voce obtera' um quadrado de lado unitario.
 Da mesma forma, se voce plotar todos os pares que
 satisfazem 'as
 exigencias do problema, voce obtera'  os dois
 triangulos internos ao
 quadrado unitario, conforme descrito na solucao.

 Repare que os tais dois triangulos sao
 simplesmente o conjunto de
 pares (x,y) capazes de definir um triangulo sobre o
 segmento unitario,
 conforme o enunciado.
 Para isso, e' necessario e suficiente que x
 e y satisfacam 'as
 seguintes condicoes:
 - o menor deles e' menor (ou igual**) que 1/2
 - o maior deles e' maior (ou igual**) que 1/2
 - a diferenca entre eles e' menor (ou igual**) que 1/2

 ** OBS: quando acontece um igual , temos um
 triangulo degenerado
 (com area zero).

 []'s
 Rogerio Ponce.



 2008/7/7 Chicao Valadares
 [EMAIL PROTECTED]:
  Os valores possiveis de x e y equivalem a area
 do quadrado unitario,
   que vale 1.
 
  Nao entendi, seria o produto xy que equivaleria a
 área?
 
 


=
Instruções para entrar na lista, sair da lista e usar a lista em
http://www.mat.puc-rio.br/~obmlistas/obm-l.html
=


[obm-l] Re: [obm-l] Re: [obm-l] Probabilidades Geomé tricas: 2 problemas difíceis

2008-07-07 Por tôpico Chicao Valadares
Os valores possiveis de x e y equivalem a area do quadrado unitario,
 que vale 1.

Nao entendi, seria o produto xy que equivaleria a área?





  Novos endereços, o Yahoo! que você conhece. Crie um email novo com a sua 
cara @ymail.com ou @rocketmail.com.
http://br.new.mail.yahoo.com/addresses

=
Instruções para entrar na lista, sair da lista e usar a lista em
http://www.mat.puc-rio.br/~obmlistas/obm-l.html
=


Re: [obm-l] Re: [obm-l] Re: [obm-l] Probabilidades Geométricas: 2 problemas difíceis

2008-07-07 Por tôpico Rogerio Ponce
Ola' Chicao,
sem perda de generalidade, eu assumi que o segmento de reta do
problema seria o segmento unitario [0 1], de forma que x pode ser
qualquer real no intervalo [0, 1].
E para cada valor de x, o ponto y tambem pode estar em qualquer
posicao no intervalo [0, 1].
Assim, usando o espaco cartesiano para plotar todos os pares (x,y)
possiveis, voce obtera' um quadrado de lado unitario.
Da mesma forma, se voce plotar todos os pares que satisfazem 'as
exigencias do problema, voce obtera'  os dois triangulos internos ao
quadrado unitario, conforme descrito na solucao.

Repare que os tais dois triangulos sao simplesmente o conjunto de
pares (x,y) capazes de definir um triangulo sobre o segmento unitario,
conforme o enunciado.
Para isso, e' necessario e suficiente que x e y satisfacam 'as
seguintes condicoes:
- o menor deles e' menor (ou igual**) que 1/2
- o maior deles e' maior (ou igual**) que 1/2
- a diferenca entre eles e' menor (ou igual**) que 1/2

** OBS: quando acontece um igual , temos um triangulo degenerado
(com area zero).

[]'s
Rogerio Ponce.



2008/7/7 Chicao Valadares [EMAIL PROTECTED]:
 Os valores possiveis de x e y equivalem a area do quadrado unitario,
  que vale 1.

 Nao entendi, seria o produto xy que equivaleria a área?



=
Instruções para entrar na lista, sair da lista e usar a lista em
http://www.mat.puc-rio.br/~obmlistas/obm-l.html
=


Re: [obm-l] Re: [obm-l] Probabilidades Geométricas: 2 problemas difíceis

2008-07-06 Por tôpico Rogerio Ponce
Ola' Chicao e colegas da lista,
considerando 2 pontos de coordenadas x e y, com distribuicao
uniforme de probabilidade sobre o segmento unitario [0,1], temos o
seguinte (a respeito de x e y):

Os valores possiveis de x e y equivalem 'a area do quadrado unitario,
que vale 1.

Reparem que, para formar um triangulo, quando x1/2 , o valor minimo
de y seria 1/2, e o maximo seria 1-x.

E quando x1/2 , o valor maximo de y seria 1/2, e o minimo seria 1-x.

Assim, os valores favoraveis de x e y equivalem 'a soma das areas dos
triangulos (0,1/2) (1/2,1) (1/2, 1/2) e (1/2, 1/2) (1/2, 0) (1, 1/2),
que vale 1/8 + 1/8 = 1/4.

Portanto, a probabilidade de formarmos um triangulo e' (1/4) / (1) = 1/4.
[]'s
Rogerio Ponce


Em 04/07/08, Chicao Valadares[EMAIL PROTECTED] escreveu:
 existe tambem um problema interessante:

 Calcule a probabilidade de dado um segmento de reta, sortear-se dois pontos
 pertencentes a esse segmento e os 3 subsegmentos formados formarem os lados
 de um triangulo.

=
Instruções para entrar na lista, sair da lista e usar a lista em
http://www.mat.puc-rio.br/~obmlistas/obm-l.html
=


Re: [obm-l] Re: [obm-l] Probabilidades Geométricas: 2 problemas difíceis

2008-07-06 Por tôpico Rogerio Ponce
Corrigindo a ultima mensagem:
...quando x1/2 , o valor maximo de y seria 1/2, e o minimo seria  x-1/2.

[]'s
Rogerio Ponce


Em 06/07/08, Rogerio Ponce[EMAIL PROTECTED] escreveu:
 Ola' Chicao e colegas da lista,
 considerando 2 pontos de coordenadas x e y, com distribuicao
 uniforme de probabilidade sobre o segmento unitario [0,1], temos o
 seguinte (a respeito de x e y):

 Os valores possiveis de x e y equivalem 'a area do quadrado unitario,
 que vale 1.

 Reparem que, para formar um triangulo, quando x1/2 , o valor minimo
 de y seria 1/2, e o maximo seria 1-x.

 E quando x1/2 , o valor maximo de y seria 1/2, e o minimo seria 1-x.

 Assim, os valores favoraveis de x e y equivalem 'a soma das areas dos
 triangulos (0,1/2) (1/2,1) (1/2, 1/2) e (1/2, 1/2) (1/2, 0) (1, 1/2),
 que vale 1/8 + 1/8 = 1/4.

 Portanto, a probabilidade de formarmos um triangulo e' (1/4) / (1) = 1/4.
 []'s
 Rogerio Ponce


 Em 04/07/08, Chicao Valadares[EMAIL PROTECTED] escreveu:
 existe tambem um problema interessante:

 Calcule a probabilidade de dado um segmento de reta, sortear-se dois
 pontos
 pertencentes a esse segmento e os 3 subsegmentos formados formarem os
 lados
 de um triangulo.


=
Instruções para entrar na lista, sair da lista e usar a lista em
http://www.mat.puc-rio.br/~obmlistas/obm-l.html
=


[obm-l] Re: [obm-l] Probabilidades Geométricas: 2 pr oblemas difíceis

2008-07-04 Por tôpico Chicao Valadares
existe tambem um problema interessante:

Calcule a probabilidade de dado um segmento de reta, sortear-se dois pontos 
pertencentes a esse segmento e os 3 subsegmentos formados formarem os lados de 
um triangulo.



O Binômio de Newton é tão belo como a Vênus de Milo.
O que há é pouca gente para dar por isso... 
Fernando Pessoa - Poesias de Alvaro Campos

_
As informações existentes nessa mensagem e no(s) arquivo(s) anexado(s) 
são
para uso restrito, sendo seu sigilo protegido por lei. Caso não seja
destinatário, saiba que leitura, divulgação ou cópia são proibidas. 
Favor
apagar as informações e notificar o remetente. O uso impróprio será 
tratado
conforme as normas da empresa e a legislação em vigor. Agradecemos sua
colaboração.


The information mentioned in this message and in the archives attached 
are
of restricted use, and its privacy is protected by law. If you are not 
the
addressee, be aware that reading, disclosure or copy are forbidden. 
Please
delete this information and notify the sender. Inappropriate use will 
be
tracted according to company's rules and valid laws. Thank you for your
cooperation.


--- Em sáb, 28/6/08, Bouskela [EMAIL PROTECTED] escreveu:

 De: Bouskela [EMAIL PROTECTED]
 Assunto: [obm-l] Probabilidades Geométricas: 2 problemas difíceis
 Para: obm-l@mat.puc-rio.br
 Data: Sábado, 28 de Junho de 2008, 10:41
 1º Problema - este é MUITO difícil!
 
  
 
 Considere uma caixa de base quadrada, cujos lados (da base)
 são unitários.
 Na base desta caixa, são traçados dois segmentos de reta:
 
 1) A própria diagonal da base; e
 
 2) O segmento de reta entre os pontos médios de dois
 lados opostos.
 
  
 
 Toma-se uma agulha de comprimento também unitário e
 joga-se, aleatoriamente,
 dentro da caixa.
 
  
 
 Pergunta-se:
 
  
 
 Qual é a probabilidade da agulha, então pousada
 horizontalmente na base da
 caixa (por hipótese!), interceptar (em um ponto qualquer)
 o segmento de reta
 de número “1”, descrito acima? E o de número “2”?
 
  
 
 Veja um problema análogo (mas, mais fácil!) em:
 
 
 http://www.cut-the-knot.com/fta/Buffon/buffon9.html
 http://www.cut-the-knot.com/fta/Buffon/buffon9.html
  
  
 2º Problema - este também é difícil, mas não tanto
 quanto o primeiro.
  
 Considere um triângulo eqüilátero. Calcule a
 probabilidade de um segmento de
 reta, determinado por um ponto qualquer de um dos lados
 desse triângulo e
 por outro ponto qualquer de um dos outros dois lados
 adjacentes, ser maior
 do que a altura do triângulo.
 
  
 
 Paradoxo de Bertrand (Bertrand's Paradox): “Given a
 circle. Find the
 probability that a chord chosen at random be longer than
 the side of an
 inscribed equilateral triangle”.
 
 Referência na Internet: 
 http://www.cut-the-knot.com/bertrand.html
 http://www.cut-the-knot.com/bertrand.html


  Novos endereços, o Yahoo! que você conhece. Crie um email novo com a sua 
cara @ymail.com ou @rocketmail.com.
http://br.new.mail.yahoo.com/addresses

=
Instru��es para entrar na lista, sair da lista e usar a lista em
http://www.mat.puc-rio.br/~obmlistas/obm-l.html
=


Re: [obm-l] Probabilidades Geométricas: 2 problemas difíceis

2008-06-30 Por tôpico Marcelo Salhab Brogliato
Olá,
estou tentando a seguinte abordagem:
Seja f(x, y, theta) uma função que é igual a 1 se a agulha com extremo no
ponto (x,y) e ângulo theta em relação ao eixo das abscissas tocar na
diagonal. E é 0 nos outros casos (quando não toca, ou quando a agulha
estiver fora do quadrado).
Seja g(x, y, theta) uma função que é igual a 1 se a agulha está dentro do
quadrado, e 0 se ela estiver fora do quadrado.
A probabilidade desejada é:
[ int{-inf ... +inf} int{-inf ... +inf} int {0 ... 2pi} f(x, y, theta)
d(theta) dy dx ] / [ int{-inf ... +inf} int{-inf ... +inf} int {0 ... 2pi}
g(x, y, theta) d(theta) dy dx ]

naturalmente, temos que se x  1 ou x  0 ou y  1 ou y  0, a agulha estará
fora do quadrado. Portanto, podemos reduzir os intervalos de 0 a 1... mais
que isso, se conseguirmos relacionar x, y e theta de modo que pegue todos os
pontos em que a agulha está no quadrado, basta integrarmos dentro desta
região.
As simplificações iniciais são:
[ int{0 ... 1} int{0 ... 1} int {0 ... 2pi} f(x, y, theta) d(theta) dy dx ]
/ [ int{0 ... 1} int{0 ... 1} int {0 ... 2pi} g(x, y, theta) d(theta) dy dx
]

seja z = cis(theta) a nossa agulha na origem.
e seja p = x + yi o ponto do extrema de nossa agulha.
p+z = (x+cos(theta)) + (y+sen(theta))i é o outro extremo da nossa agulha.
Para determinarmos se a agulha toca ou não na diagonal, temos 2 casos:
i) x  y  e  x+cos(theta)  y+sen(theta)
ii) x  y  e  x+cos(theta)  y+sen(theta)

Para determinar a região em que a agulha está dentro do quadrado, temos:
Re(p+z) = 1 e Im(p+z) = 1
isto é:
x + cos(r) = 1  e  y + sen(r) = 1

ainda não cheguei a nenhuma conclusão... analisei a região em um programa
gráfico e estou tentando encontrar uma equação fechada para ela (para
integrarmos nessa região... e passarmos a nos preocupar somente com a agulha
tocar ou nao a diagonal).

abraços,
Salhab




2008/6/28 Bouskela [EMAIL PROTECTED]:

  1º Problema - este é MUITO difícil!



 Considere uma caixa de base quadrada, cujos lados (da base) são unitários.
 Na base desta caixa, são traçados dois segmentos de reta:

 1) A própria diagonal da base; e

 2) O segmento de reta entre os pontos médios de dois lados opostos.



 Toma-se uma agulha de comprimento também unitário e joga-se,
 aleatoriamente, dentro da caixa.



 Pergunta-se:



 Qual é a probabilidade da agulha, então pousada horizontalmente na base da
 caixa (por hipótese!), interceptar (em um ponto qualquer) o segmento de reta
 de número 1, descrito acima? E o de número 2?



 Veja um problema análogo (mas, mais fácil!) em:
 http://www.cut-the-knot.com/fta/Buffon/buffon9.html


 2º Problema - este também é difícil, mas não tanto quanto o primeiro.


 Considere um triângulo eqüilátero. Calcule a probabilidade de um segmento
 de reta, determinado por um ponto qualquer de um dos lados desse triângulo e
 por outro ponto qualquer de um dos outros dois lados adjacentes, ser maior
 do que a altura do triângulo.



 Paradoxo de Bertrand (Bertrand's Paradox): Given a circle. Find the
 probability that a chord chosen at random be longer than the side of an
 inscribed equilateral triangle.

 Referência na Internet: http://www.cut-the-knot.com/bertrand.html



Re: [obm-l] Probabilidades Geométricas: 2 problemas difíceis

2008-06-30 Por tôpico Marcelo Salhab Brogliato
Opa,
acho que consegui determinar a região... vamos lá:
0 = x = 1
0 = y = 1
0 = x + cos(theta) = 1
0 = y + sen(theta) = 1

logo:
0 = x = 1
0 = y = 1
-cos(theta) = x = 1 - cos(theta)
-sen(theta) = y = 1 - sen(theta)

portanto, podemos escrever nossas integrais do seguinte modo:
int {0 ... 2pi} int { 1-cos(theta) ... max(-cos(theta), 0) } int { 1 -
sen(theta) ... max( -sen(theta), 0) } dy dx dtheta
veja que nesta região, g(x, y, theta) = 1... portanto, basta integrarmos
mesmo!
Falta apenas determinarmos f(x, y, theta).. o que não está parecendo muito
difícil.

Antes, vamos apenas dividir esta integral em 4, para retirarmos os max...

int {0 ... pi/2} int {1-cos(theta) ... 0} int {1-sen(theta) ... 0} dy dx
d(theta) +
int {pi/2 ... pi} int {1-cos(theta) ... -cos(theta)} int {1-sen(theta) ...
0} dy dx d(theta) +
int {pi ... 3pi/2} int {1-cos(theta) ... -cos(theta)} int {1-sen(theta) ...
-sen(theta)} dy dx d(theta) +
int {3pi/2 ... 2pi} int {1-cos(theta) ... 0} int {1-sen(theta) ...
-sen(theta)} dy dx d(theta)

resolvendo, temos:
(pi/2 - 3/2) + (pi/2 - 1) + (pi/2) + (1 + pi/2) = 2pi - 3/2
[posso ter errado conta..]

vou pensar no f agora.. acredito que seja apenas dividir mais ainda nossas
integrais...
assim que concluir algo mando outra mensagem..

abraços,
Salhab



2008/6/30 Marcelo Salhab Brogliato [EMAIL PROTECTED]:

 Olá,
 estou tentando a seguinte abordagem:
 Seja f(x, y, theta) uma função que é igual a 1 se a agulha com extremo no
 ponto (x,y) e ângulo theta em relação ao eixo das abscissas tocar na
 diagonal. E é 0 nos outros casos (quando não toca, ou quando a agulha
 estiver fora do quadrado).
 Seja g(x, y, theta) uma função que é igual a 1 se a agulha está dentro do
 quadrado, e 0 se ela estiver fora do quadrado.
 A probabilidade desejada é:
 [ int{-inf ... +inf} int{-inf ... +inf} int {0 ... 2pi} f(x, y, theta)
 d(theta) dy dx ] / [ int{-inf ... +inf} int{-inf ... +inf} int {0 ... 2pi}
 g(x, y, theta) d(theta) dy dx ]

 naturalmente, temos que se x  1 ou x  0 ou y  1 ou y  0, a agulha
 estará fora do quadrado. Portanto, podemos reduzir os intervalos de 0 a 1...
 mais que isso, se conseguirmos relacionar x, y e theta de modo que pegue
 todos os pontos em que a agulha está no quadrado, basta integrarmos dentro
 desta região.
 As simplificações iniciais são:
 [ int{0 ... 1} int{0 ... 1} int {0 ... 2pi} f(x, y, theta) d(theta) dy dx ]
 / [ int{0 ... 1} int{0 ... 1} int {0 ... 2pi} g(x, y, theta) d(theta) dy dx
 ]

 seja z = cis(theta) a nossa agulha na origem.
 e seja p = x + yi o ponto do extrema de nossa agulha.
 p+z = (x+cos(theta)) + (y+sen(theta))i é o outro extremo da nossa agulha.
 Para determinarmos se a agulha toca ou não na diagonal, temos 2 casos:
 i) x  y  e  x+cos(theta)  y+sen(theta)
 ii) x  y  e  x+cos(theta)  y+sen(theta)

 Para determinar a região em que a agulha está dentro do quadrado, temos:
 Re(p+z) = 1 e Im(p+z) = 1
 isto é:
 x + cos(r) = 1  e  y + sen(r) = 1

 ainda não cheguei a nenhuma conclusão... analisei a região em um programa
 gráfico e estou tentando encontrar uma equação fechada para ela (para
 integrarmos nessa região... e passarmos a nos preocupar somente com a agulha
 tocar ou nao a diagonal).

 abraços,
 Salhab




 2008/6/28 Bouskela [EMAIL PROTECTED]:

  1º Problema - este é MUITO difícil!



 Considere uma caixa de base quadrada, cujos lados (da base) são unitários.
 Na base desta caixa, são traçados dois segmentos de reta:

 1) A própria diagonal da base; e

 2) O segmento de reta entre os pontos médios de dois lados opostos.



 Toma-se uma agulha de comprimento também unitário e joga-se,
 aleatoriamente, dentro da caixa.



 Pergunta-se:



 Qual é a probabilidade da agulha, então pousada horizontalmente na base da
 caixa (por hipótese!), interceptar (em um ponto qualquer) o segmento de reta
 de número 1, descrito acima? E o de número 2?



 Veja um problema análogo (mas, mais fácil!) em:
 http://www.cut-the-knot.com/fta/Buffon/buffon9.html


 2º Problema - este também é difícil, mas não tanto quanto o primeiro.


 Considere um triângulo eqüilátero. Calcule a probabilidade de um segmento
 de reta, determinado por um ponto qualquer de um dos lados desse triângulo e
 por outro ponto qualquer de um dos outros dois lados adjacentes, ser maior
 do que a altura do triângulo.



 Paradoxo de Bertrand (Bertrand's Paradox): Given a circle. Find the
 probability that a chord chosen at random be longer than the side of an
 inscribed equilateral triangle.

 Referência na Internet: http://www.cut-the-knot.com/bertrand.html





Re: [obm-l] Probabilidades Geométricas: 2 problemas difíceis

2008-06-30 Por tôpico Bouskela
Salhab, saudações!

1º - Enviei-lhe uma mensagem, apontando que, em relação ao problema
concernente à eq. x^2 - xy + y^2 = Cte , é necessário fazer alguns ajustes
na sua solução qdo. uma das raízes é igual a 0:

P.ex., se a=0 , então o par (-a, -b) é igual ao par (a, a-b) .

2º - Quanto a este problema de Probabilidades Geométricas, acredito que vc.
esteja indo por um caminho correto, mas tortuoso! É mais simples criar
faixas infinitesimais, paralelas a um dos lados da base e à linha que divide
a base em 2 áreas iguais; fazer com que uma das extremidades da agulha caia
nestas faixas infinitesimais; verificar qual é a condição de contorno em que
a agulha não intercepta a linha supracitada; dividir o range desta
condição de contorno pela condição de contorno possível (a agulha está
pousada horizontalmente sobre a base); integrar e...

Sds.,
AB!

2008/6/30 Marcelo Salhab Brogliato [EMAIL PROTECTED]:

 Opa,
 acho que consegui determinar a região... vamos lá:
 0 = x = 1
 0 = y = 1
 0 = x + cos(theta) = 1
 0 = y + sen(theta) = 1

 logo:
 0 = x = 1
 0 = y = 1
 -cos(theta) = x = 1 - cos(theta)
 -sen(theta) = y = 1 - sen(theta)

 portanto, podemos escrever nossas integrais do seguinte modo:
 int {0 ... 2pi} int { 1-cos(theta) ... max(-cos(theta), 0) } int { 1 -
 sen(theta) ... max( -sen(theta), 0) } dy dx dtheta
 veja que nesta região, g(x, y, theta) = 1... portanto, basta integrarmos
 mesmo!
 Falta apenas determinarmos f(x, y, theta).. o que não está parecendo muito
 difícil.

 Antes, vamos apenas dividir esta integral em 4, para retirarmos os max...

 int {0 ... pi/2} int {1-cos(theta) ... 0} int {1-sen(theta) ... 0} dy dx
 d(theta) +
 int {pi/2 ... pi} int {1-cos(theta) ... -cos(theta)} int {1-sen(theta) ...
 0} dy dx d(theta) +
 int {pi ... 3pi/2} int {1-cos(theta) ... -cos(theta)} int {1-sen(theta) ...
 -sen(theta)} dy dx d(theta) +
 int {3pi/2 ... 2pi} int {1-cos(theta) ... 0} int {1-sen(theta) ...
 -sen(theta)} dy dx d(theta)

 resolvendo, temos:
 (pi/2 - 3/2) + (pi/2 - 1) + (pi/2) + (1 + pi/2) = 2pi - 3/2
 [posso ter errado conta..]

 vou pensar no f agora.. acredito que seja apenas dividir mais ainda nossas
 integrais...
 assim que concluir algo mando outra mensagem..

 abraços,
 Salhab



 2008/6/30 Marcelo Salhab Brogliato [EMAIL PROTECTED]:

 Olá,
 estou tentando a seguinte abordagem:
 Seja f(x, y, theta) uma função que é igual a 1 se a agulha com extremo no
 ponto (x,y) e ângulo theta em relação ao eixo das abscissas tocar na
 diagonal. E é 0 nos outros casos (quando não toca, ou quando a agulha
 estiver fora do quadrado).
 Seja g(x, y, theta) uma função que é igual a 1 se a agulha está dentro do
 quadrado, e 0 se ela estiver fora do quadrado.
 A probabilidade desejada é:
 [ int{-inf ... +inf} int{-inf ... +inf} int {0 ... 2pi} f(x, y, theta)
 d(theta) dy dx ] / [ int{-inf ... +inf} int{-inf ... +inf} int {0 ... 2pi}
 g(x, y, theta) d(theta) dy dx ]

 naturalmente, temos que se x  1 ou x  0 ou y  1 ou y  0, a agulha
 estará fora do quadrado. Portanto, podemos reduzir os intervalos de 0 a 1...
 mais que isso, se conseguirmos relacionar x, y e theta de modo que pegue
 todos os pontos em que a agulha está no quadrado, basta integrarmos dentro
 desta região.
 As simplificações iniciais são:
 [ int{0 ... 1} int{0 ... 1} int {0 ... 2pi} f(x, y, theta) d(theta) dy dx
 ] / [ int{0 ... 1} int{0 ... 1} int {0 ... 2pi} g(x, y, theta) d(theta) dy
 dx ]

 seja z = cis(theta) a nossa agulha na origem.
 e seja p = x + yi o ponto do extrema de nossa agulha.
 p+z = (x+cos(theta)) + (y+sen(theta))i é o outro extremo da nossa agulha.
 Para determinarmos se a agulha toca ou não na diagonal, temos 2 casos:
 i) x  y  e  x+cos(theta)  y+sen(theta)
 ii) x  y  e  x+cos(theta)  y+sen(theta)

 Para determinar a região em que a agulha está dentro do quadrado, temos:
 Re(p+z) = 1 e Im(p+z) = 1
 isto é:
 x + cos(r) = 1  e  y + sen(r) = 1

 ainda não cheguei a nenhuma conclusão... analisei a região em um programa
 gráfico e estou tentando encontrar uma equação fechada para ela (para
 integrarmos nessa região... e passarmos a nos preocupar somente com a agulha
 tocar ou nao a diagonal).

 abraços,
 Salhab




 2008/6/28 Bouskela [EMAIL PROTECTED]:

   1º Problema - este é MUITO difícil!



 Considere uma caixa de base quadrada, cujos lados (da base) são
 unitários. Na base desta caixa, são traçados dois segmentos de reta:

 1) A própria diagonal da base; e

 2) O segmento de reta entre os pontos médios de dois lados opostos.



 Toma-se uma agulha de comprimento também unitário e joga-se,
 aleatoriamente, dentro da caixa.



 Pergunta-se:



 Qual é a probabilidade da agulha, então pousada horizontalmente na base
 da caixa (por hipótese!), interceptar (em um ponto qualquer) o segmento de
 reta de número 1, descrito acima? E o de número 2?



 Veja um problema análogo (mas, mais fácil!) em:
 http://www.cut-the-knot.com/fta/Buffon/buffon9.html


 2º Problema - este também é difícil, mas não tanto quanto o primeiro.


 

[obm-l] Probabilidades Geométricas: 2 problemas difíceis

2008-06-28 Por tôpico Bouskela
1º Problema - este é MUITO difícil!

 

Considere uma caixa de base quadrada, cujos lados (da base) são unitários.
Na base desta caixa, são traçados dois segmentos de reta:

1) A própria diagonal da base; e

2) O segmento de reta entre os pontos médios de dois lados opostos.

 

Toma-se uma agulha de comprimento também unitário e joga-se, aleatoriamente,
dentro da caixa.

 

Pergunta-se:

 

Qual é a probabilidade da agulha, então pousada horizontalmente na base da
caixa (por hipótese!), interceptar (em um ponto qualquer) o segmento de reta
de número “1”, descrito acima? E o de número “2”?

 

Veja um problema análogo (mas, mais fácil!) em:

 http://www.cut-the-knot.com/fta/Buffon/buffon9.html
http://www.cut-the-knot.com/fta/Buffon/buffon9.html
 
 
2º Problema - este também é difícil, mas não tanto quanto o primeiro.
 
Considere um triângulo eqüilátero. Calcule a probabilidade de um segmento de
reta, determinado por um ponto qualquer de um dos lados desse triângulo e
por outro ponto qualquer de um dos outros dois lados adjacentes, ser maior
do que a altura do triângulo.

 

Paradoxo de Bertrand (Bertrand's Paradox): “Given a circle. Find the
probability that a chord chosen at random be longer than the side of an
inscribed equilateral triangle”.

Referência na Internet:  http://www.cut-the-knot.com/bertrand.html
http://www.cut-the-knot.com/bertrand.html



[obm-l] Probabilidades Geométricas: 2 problemas difíceis

2008-06-28 Por tôpico Bouskela
1º Problema - este é MUITO difícil!



Considere uma caixa de base quadrada, cujos lados (da base) são unitários.
Na base desta caixa, são traçados dois segmentos de reta:

1) A própria diagonal da base; e

2) O segmento de reta entre os pontos médios de dois lados opostos.



Toma-se uma agulha de comprimento também unitário e joga-se, aleatoriamente,
dentro da caixa.



Pergunta-se:



Qual é a probabilidade da agulha, então pousada horizontalmente na base da
caixa (por hipótese!), interceptar (em um ponto qualquer) o segmento de reta
de número 1, descrito acima? E o de número 2?



Veja um problema análogo (mas, mais fácil!) em:
http://www.cut-the-knot.com/fta/Buffon/buffon9.html


2º Problema - este também é difícil, mas não tanto quanto o primeiro.


Considere um triângulo eqüilátero. Calcule a probabilidade de um segmento de
reta, determinado por um ponto qualquer de um dos lados desse triângulo e
por outro ponto qualquer de um dos outros dois lados adjacentes, ser maior
do que a altura do triângulo.



Paradoxo de Bertrand (Bertrand's Paradox): Given a circle. Find the
probability that a chord chosen at random be longer than the side of an
inscribed equilateral triangle.

Referência na Internet: http://www.cut-the-knot.com/bertrand.html


Re: [obm-l] Probabilidades e o Primeiro Lema de Kaplansky

2008-03-23 Por tôpico alexmay nunes soares
Ulysses, acredito que ao dizer um par não esteja sendo excluída a 
possibilidade de haver mais de um par, certamente que se fosse dito pelo menos 
um par teríamos entendido de imediato a solicitação, mas, na minha opnião, 
dizer haja um par é o mesmo que dizer haja pelo menos um par, seria 
diferente se ele tivesse amarrado com haja extamente um par, ou haja 
apenas... ou haja somente... etc. Ademais devemos lembrar que qdo tratamos 
com conjuntos agimos de forma semelhante, pois ao dizer q x é elemento de A 
estamos considerando a possibilidade de ele ser elemento de B tb, e qdo 
queremos nos certificar do contrário dizemos x é elemento apenas de A
   
  Ulysses Coelho de Souza Jr. [EMAIL PROTECTED] escreveu:
  Olá a todos,
   
  A questão abaixo é de um vestibular recente. Acredito que o examinador quis 
dizer pelo menos um par ao invés de um par.
   
  Comentários serão bem-vindos. 
   
  No Concurso da Mega-Sena são sorteados
  6 números de 01 a 60. Por exemplo, o concurso
  924 teve como números sorteados
  02,20,21,27,51 e 60, ou seja, houve um par de
  números consecutivos, 20 e 21. A probabilidade
  de que no jogo da Mega-Sena haja um par de
  números consecutivos sorteados é:
   
  (A) 54!/60!
  (B) 53!/59!
  (C) 1-(56!55!)/(49!60!)
  (D) 1-(54!53!)/(48!60!)
  (E) 1-(55!54!)/(49!60!)
   
  Um abraço,
   
  Ulysses C. de Souza.


   
-
Abra sua conta no Yahoo! Mail, o único sem limite de espaço para armazenamento! 

Res: [obm-l] Probabilidades e o Primeiro Lema de Kaplansky

2008-03-23 Por tôpico Eduardo Estrada
Façamos o seguinte, Ulysses:

Queremos que haja pelo menos um par de números consecutivos sorteados. Então 
vamos contar os sorteios que não contêm números consecutivos. Para tanto, 
consideremos seqüências de 60 dígitos formadas por 54 0's e 6 1's, de tal 
maneira que, se o i-ésimo dígito for 0, então o número i não foi sorteado e, 
caso cotrário, foi sorteado. Por exemplo:

10001000.001

Na seqüência acima, foram sorteados os números 5, 9, 60 etc., pois essas 
posições são ocupadas por 1's. Assim, se imaginarmos os 54 0's emparelhados, 
temos:

_0_0_0_0_0_..._0_0_0_

Onde os 55 traços _ indicam posições candidatas a serem ocupadas por 6 1's, ou 
seja, definem os números sorteados. Logo, podemos selecioná-las de C(55,6) 
maneiras. Como o total de sorteios é C(60,6), segue que a probabilidade de não 
haver números consecutivos é C(55,6)/C(60,6). Portanto, a probabilidade de 
haver números consecutivos é:

1-C(55,6)/C(60,6) que, após algumas manipulações, nos leva à alternativa E.

Um abraço,
Eduardo Estrada

- Mensagem original 
De: Ulysses Coelho de Souza Jr. [EMAIL PROTECTED]
Para: obm-l@mat.puc-rio.br
Enviadas: Sábado, 22 de Março de 2008 20:58:56
Assunto: [obm-l] Probabilidades e o Primeiro Lema de Kaplansky

  
Olá a todos,
 
A questão abaixo é de um vestibular recente. Acredito 
que o examinador quis dizer pelo menos um par ao invés de um 
par.
 
Comentários serão bem-vindos. 
 
No Concurso da Mega-Sena são sorteados


 
6 números de 01 a 60. Por exemplo, o 
concurso
 
924 teve como números 
sorteados
 
02,20,21,27,51 e 60, ou seja, houve um par 
de
 
números consecutivos, 20 e 21. A 
probabilidade
 
de que no jogo da Mega-Sena haja um par 
de
 
números consecutivos sorteados 
é:
 
 
 
(A) 
54!/60!
 
(B) 
53!/59!
 
(C) 
1-(56!55!)/(49!60!)
 
(D) 
1-(54!53!)/(48!60!)
 
(E) 
1-(55!54!)/(49!60!)
 
Um abraço,
 
Ulysses C. de 
Souza.






  Abra sua conta no Yahoo! Mail, o único sem limite de espaço para 
armazenamento!
http://br.mail.yahoo.com/

[obm-l] Probabilidades e o Primeiro Lema de Kaplansky

2008-03-22 Por tôpico Ulysses Coelho de Souza Jr.
Olá a todos,



A questão abaixo é de um vestibular recente. Acredito que o examinador quis 
dizer pelo menos um par ao invés de um par.



Comentários serão bem-vindos. 



No Concurso da Mega-Sena são sorteados

6 números de 01 a 60. Por exemplo, o concurso

924 teve como números sorteados

02,20,21,27,51 e 60, ou seja, houve um par de

números consecutivos, 20 e 21. A probabilidade

de que no jogo da Mega-Sena haja um par de

números consecutivos sorteados é:

 

(A) 54!/60!

(B) 53!/59!

(C) 1-(56!55!)/(49!60!)

(D) 1-(54!53!)/(48!60!)

(E) 1-(55!54!)/(49!60!)

Um abraço,

Ulysses C. de Souza.

RE: [obm-l] probabilidades

2007-12-05 Por tôpico Pedro Cardoso


Opa, Ralph.
 
Eu já tinha lido outros e-mails seus com essa advertência, mas só agora percebi 
que nesse caso* seu alerta também era válido. Sou um dos alunos novos que 
aprenderam assim - que probabilidade é caso favorável/caso possível. Vou até 
testar meu professor!
 
*me refiro ao e-mail do ralph sobre o problema de probabilidade enviado pelo 
crmoraes, sobre sorteio de bolinha de gude.
 
Enfim, muito obrigado (pelo elogio e pela aula).
_
Veja mapas e encontre as melhores rotas para fugir do trânsito com o Live 
Search Maps!
http://www.livemaps.com.br/index.aspx?tr=true

[obm-l] probabilidades

2007-12-04 Por tôpico crmoraes
Podem me ajudar com esses problemas?

I-)Tenho o mesmo número de bolinhas de gude verdes, amarelas, azuis e
brancas.
1. Qual a probabilidade de, em 10 bolinhas, não ter as 4 cores?


II-) Tenho o mesmo número de bolinhas de gude azuis, vermelhas e amarelas.
1. Qual a probabilidade de, em 18 bolinhas, todas serem da mesma cor?
2. Qual a probabilidade de, em 18 bolinhas, serem apenas de 2 cores
(quaisquer)?
3. Qual a probabilidade de, em 18 bolinhas, ter pelo menos 1 de cada cor?


-- 
Mensagem verificada contra virus. 
Provedor Claretianas. 

=
Instruções para entrar na lista, sair da lista e usar a lista em
http://www.mat.puc-rio.br/~obmlistas/obm-l.html
=


Re: [obm-l] probabilidades

2007-12-04 Por tôpico Ralph Teixeira
I) Tecnicamente, isto depende do número de bolinhas que você tem. Se você
tiver 3 bolinhas de cada cor, por exemplo, em 10 bolinhas sorteadas sem
reposição você tem 100% de probabilidade de ter as 4 cores!

Vou interpretar de outro jeito (que é equivalente a tomar o número de
bolinhas indo para infinito, ou supor que as bolinhas são sorteadas **com**
reposição): vou fazer 10 sorteios independentes; cada sorteio consiste
em escolher uma de 4 cores, todas com a mesma probabilidade 1/4. A pergunta
é: qual a chance de faltar alguma cor no decorrer dos 10 sorteios?

Vou usar M (aMarelo), Z (aZul), V (verde) e B (branco) para denotar o número
de vezes que cada cor apareceu nos 10 sorteio. Note que M+Z+V+B=10. Queremos
Pr(M=0 ou Z=0 ou V=0 ou B=0).

i) Pr(M=0)=Pr(Z=0)=Pr(V=0)=Pr(B=0)=(3/4)^10
(3/4 de chance em cada sorteio daquela cor específica não aparecer)

ii) Pr(M=Z=0)=Pr(M=V=0)=...=Pr(B=V=0)=(2/4)^10
(2/4 de chance em cada sorteio de ambas aquelas cores não aparecerem)

iii) Pr(M=Z=V=0)=Pr(M=Z=B=0)=...=(1/4)^10
(basicamente, cada um destes significa tudo de uma cor só)

Pelo princípio da inclusão-exclusão, queremos
Pr(M=0 ou Z=0 ou V=0 ou B=0) =
= Pr(M=0)+Pr(Z=0)+...+Pr(B=0)
-(Pr(M=0 e Z=0)+Pr(M=0 e V=0)+...Pr(B=V=0))
+Pr(M=Z=V=0)+Pr(M=Z=B=0)+...
-Pr(M=Z=V=B=0) =
= 4.(3/4)^10-6.(2/4)^10+4.(1/4)^10 - 0 = 230056/(2^20) = 21.94%

II) De novo, só dá para achar um número se a gente supuser que as bolinhas
são sorteadas COM reposição (que é equivalente ao número de bolinhas tender
a infinito). O método é igual ao de ali em cima: usarei a mesma notação Z (#
de aZuis), V (# de Vermelhas) e M (# de aMarelas). Note que Z+V+M=18.

i) Pr(Z=18)=Pr(V=18)=Pr(M=18)=(1/3)^18
Então Pr(todas da mesma cor) = 3.(1/3)^18 - 1/9(3^17) = 7.74(10^-9)

ii) Pr(Z=0)=Pr(V=0)=Pr(M=0)=(2/3)^18
Então Pr(Z=0 ou V=0 ou M=0) =
= Pr(Z=0)+Pr(V=0)+Pr(M=0)
-Pr(Z=V=0)-Pr(Z=M=0)-Pr(V=M=0)
+Pr(Z=V=M=0) =
= 3.(2/3)^18-3.(1/3)^18+0 = 0.203%

iii) Queremos Pr(Z0 e V0 e M0) que é exatamente o complementar do item
anterior. Então a resposta é
1-(3.(2/3)^18-3.(1/3)^18) = 99.797%


On 12/4/07, [EMAIL PROTECTED] [EMAIL PROTECTED] wrote:

 Podem me ajudar com esses problemas?

 I-)Tenho o mesmo número de bolinhas de gude verdes, amarelas, azuis e
 brancas.
 1. Qual a probabilidade de, em 10 bolinhas, não ter as 4 cores?


 II-) Tenho o mesmo número de bolinhas de gude azuis, vermelhas e amarelas.
 1. Qual a probabilidade de, em 18 bolinhas, todas serem da mesma cor?
 2. Qual a probabilidade de, em 18 bolinhas, serem apenas de 2 cores
 (quaisquer)?
 3. Qual a probabilidade de, em 18 bolinhas, ter pelo menos 1 de cada cor?


 --
 Mensagem verificada contra virus.
 Provedor Claretianas.

 =
 Instruções para entrar na lista, sair da lista e usar a lista em
 http://www.mat.puc-rio.br/~obmlistas/obm-l.html
 =



Re: [obm-l] probabilidades

2007-12-04 Por tôpico Ralph Teixeira
On 12/4/07, Ralph Teixeira [EMAIL PROTECTED] wrote:

 I) Tecnicamente, isto depende do número de bolinhas que você tem. Se você
 tiver 3 bolinhas de cada cor, por exemplo, em 10 bolinhas sorteadas sem
 reposição você tem 100% de probabilidade de ter as 4 cores!

 Vou interpretar de outro jeito (que é equivalente a tomar o número de
 bolinhas indo para infinito, ou supor que as bolinhas são sorteadas **com**
 reposição): vou fazer 10 sorteios independentes; cada sorteio consiste
 em escolher uma de 4 cores, todas com a mesma probabilidade 1/4. A pergunta
 é: qual a chance de faltar alguma cor no decorrer dos 10 sorteios?

 Vou usar M (aMarelo), Z (aZul), V (verde) e B (branco) para denotar o
 número de vezes que cada cor apareceu nos 10 sorteio. Note que M+Z+V+B=10.
 Queremos Pr(M=0 ou Z=0 ou V=0 ou B=0).

 i) Pr(M=0)=Pr(Z=0)=Pr(V=0)=Pr(B=0)=(3/4)^10
 (3/4 de chance em cada sorteio daquela cor específica não aparecer)

 ii) Pr(M=Z=0)=Pr(M=V=0)=...=Pr(B=V=0)=(2/4)^10
 (2/4 de chance em cada sorteio de ambas aquelas cores não aparecerem)

 iii) Pr(M=Z=V=0)=Pr(M=Z=B=0)=...=(1/4)^10
 (basicamente, cada um destes significa tudo de uma cor só)

 Pelo princípio da inclusão-exclusão, queremos
 Pr(M=0 ou Z=0 ou V=0 ou B=0) =
 = Pr(M=0)+Pr(Z=0)+...+Pr(B=0)
 -(Pr(M=0 e Z=0)+Pr(M=0 e V=0)+...Pr(B=V=0))
 +Pr(M=Z=V=0)+Pr(M=Z=B=0)+...
 -Pr(M=Z=V=B=0) =
 = 4.(3/4)^10-6.(2/4)^10+4.(1/4)^10 - 0 = 230056/(2^20) = 21.94%

 II) De novo, só dá para achar um número se a gente supuser que as bolinhas
 são sorteadas COM reposição (que é equivalente ao número de bolinhas tender
 a infinito). O método é igual ao de ali em cima: usarei a mesma notação Z (#
 de aZuis), V (# de Vermelhas) e M (# de aMarelas). Note que Z+V+M=18.

 i) Pr(Z=18)=Pr(V=18)=Pr(M=18)=(1/3)^18
 Então Pr(todas da mesma cor) = 3.(1/3)^18 - 1/9(3^17) = 7.74(10^-9)

 ii) Pr(Z=0)=Pr(V=0)=Pr(M=0)=(2/3)^18
 Então Pr(Z=0 ou V=0 ou M=0) =
 = Pr(Z=0)+Pr(V=0)+Pr(M=0)
 -Pr(Z=V=0)-Pr(Z=M=0)-Pr(V=M=0)
 +Pr(Z=V=M=0) =
 = 3.(2/3)^18-3.(1/3)^18


(Observação: o que eu fiz foi a probabilidade de aparecem 2 cores OU MENOS).
Se você quiser a probabilidade de aparecerem EXATAMENTE cores, tem que
subtrair de novo a probabilidade de aparecer 1 cor só:
Pr(EXATAMENTE 2 cores) = 3.(2/3)^18-3.(1/3)^18 - (3.(1/3)^18 - 1/9(3^17)).
Na prática, a diferença é pouca.)



 iii) Queremos Pr(Z0 e V0 e M0) que é exatamente o complementar do
 item anterior. Então a resposta é
 1-(3.(2/3)^18-3.(1/3)^18) = 99.797%


 On 12/4/07, [EMAIL PROTECTED] [EMAIL PROTECTED] 
 wrote:
 
  Podem me ajudar com esses problemas?
 
  I-)Tenho o mesmo número de bolinhas de gude verdes, amarelas, azuis e
  brancas.
  1. Qual a probabilidade de, em 10 bolinhas, não ter as 4 cores?
 
 
  II-) Tenho o mesmo número de bolinhas de gude azuis, vermelhas e
  amarelas.
  1. Qual a probabilidade de, em 18 bolinhas, todas serem da mesma cor?
  2. Qual a probabilidade de, em 18 bolinhas, serem apenas de 2 cores
  (quaisquer)?
  3. Qual a probabilidade de, em 18 bolinhas, ter pelo menos 1 de cada
  cor?
 
 
  --
  Mensagem verificada contra virus.
  Provedor Claretianas.
 
 
  =
  Instruções para entrar na lista, sair da lista e usar a lista em
  http://www.mat.puc-rio.br/~obmlistas/obm-l.html
 
  =
 




[obm-l] PROBABILIDADES ATÍPICAS!

2007-03-26 Por tôpico Jorge Luis Rodrigues e Silva Luis
Dalgliesh, o detetive, tem-se na conta de um juiz perspicaz da natureza 
humana. Apurou-se, através de testes adequados, que 80% das vezes em que 
afirma que um suspeito mente, acerta. Dalgliesh declara que Jones está a 
mentir. O perito do polígrafo, que acerta 100% das vezes, diz que 40% dos 
indivíduos, inquiridos por Dalgliesh dizem a verdade. Qual a probabilidade 
de Jones dizer a verdade?


Uma dona de casa tem probabilidade 0,6 de encontrar carne no açougue se no 
dia anterior existia carne, e tem probabilidade 0,30 de achar se no dia 
anterior não existia. Qual a probabilidade de não conseguir comprar carne 
daqui a 4 dias, sabendo que hoje é igualmente provável achar ou não carne?


Tendo-se tomado, ao acaso, dois números positivos x e y, que não excedam a 
dois, determinar a probabilidade de que o produto xy não exceda à unidade e 
o quociente y/x não exceda a dois.


Um dado normal é lançado repetidamente até que o primeiro total das rodadas 
exceda 12. Qual é o mais provável total que será obtido?


Abraços!

_
Seja um dos primeiros a testar o novo Windows Live Mail Beta- grátis. Acesse 
http://www.ideas.live.com/programpage.aspx?versionId=5d21c51a-b161-4314-9b0e-4911fb2b2e6d


=
Instruções para entrar na lista, sair da lista e usar a lista em
http://www.mat.puc-rio.br/~nicolau/olimp/obm-l.html
=


[obm-l] probabilidades (páscoa)

2006-04-13 Por tôpico Leo



Gostaria de saber se tem como calcuilarmos a 
probabilidade da k-ésima pessoa a tirar um papelzinho de chococulto tirar ela 
mesma num grupo d n pessoas com k=n

Grato


[obm-l] PROBABILIDADES!

2006-03-03 Por tôpico Jorge Luis Rodrigues e Silva Luis
Em uma mesma caixa há dez pares de meias brancas e dez pares de meias 
pretas, assim como dez pares de luvas brancas e dez pares de luvas pretas. 
Supondo distinção entre as luvas das mãos esquerda e direita, quais as 
probabilidades de, retirarmos no escuro um par de peças do mesmo tipo e 
mesma cor? Um par de peças do mesmo tipo e cor branca? Na certeza de ocorrer 
tais eventos, quantas retiradas terei que fazer?


Ele e Ela dizem a verdade com probabilidades iguais a 3/4 e 3/5, 
respectivamente, independente um do outro. Se Ele faz uma afirmação e Ela 
diz que Ele mente, calcular a probabilidade de que Ele diz a verdade.


Se em três faces de um dado perfeito for colocado o número 1 e nas outras 
três faces o número 6 com probabilidade 1/2. Qual o valor da média dessa 
distribuição?


Escolhido ao acaso um divisor positivo do número 60, qual é a probabilidade 
de ele ser primo?



Abraços e Bom Final de Semana!

_
Copa 2006: Já está na hora de saber o que é ‘Freundschaftsspiel’ Clique 
aqui! http://copa.br.msn.com/extra/dicionario/


=
Instruções para entrar na lista, sair da lista e usar a lista em
http://www.mat.puc-rio.br/~nicolau/olimp/obm-l.html
=


[obm-l] PROBABILIDADES CHANCE!

2006-02-23 Por tôpico Jorge Luis Rodrigues e Silva Luis
Um atleta atribui uma chance de 2 para 1, mas não de 3 para 1, de que ele 
derrotará seu companheiro em uma queda de braço. O que é que isto nos diz 
sobre a probabilidade que ele atribui à sua vitória?


Se a chance de um time de futebol americano de uma faculdade ganhar o 
próximo jogo é melhor do que 5 para 4, o que se pode dizer quanto à 
probabilidade de vencer o próximo jogo?


Há dois Porsches em uma corrida, e um repórter acha que as chances contra 
suas vitórias são de 4 para 1 e 5 para 1 respectivamente. Para ser 
consistente, que chance ele deve atribuir ao evento de nenhum dos dois 
carros ganhar?


Qual a probabilidade de Piquet vencer determinada corrida se, segundo os 
técnicos de sua escuderia as suas chances são de 9 para 7?


Abraços!

_
Copa 2006: Sabe como se diz ‘pênalti’ em alemão? Clique aqui! 
http://copa.br.msn.com/extra/dicionario/l-z/


=
Instruções para entrar na lista, sair da lista e usar a lista em
http://www.mat.puc-rio.br/~nicolau/olimp/obm-l.html
=


Re: [obm-l] PROBABILIDADES CHANCE!

2006-02-23 Por tôpico João Luís Gomes Guimarães

Minhas sugestões de soluções:

Inicialmente, um raciocínio que vale para todos: uma chance de vitória de 2 
para 1, por exemplo, quer dizer que a cada 3 contendas teremos 2 vitórias e 
uma derrota. Vamos lá:


1) Se a chance é 2 para 1, isso equivale a 2 vitórias em 3 quedas, então a 
probabilidade é 2/3, 67% aproximadamente


2) Mesmo raciocínio: 5 para 4 equivale a 5/9, ou 56% aproximadamente

3) (4/5)x(5/6) = 2/3. Dá o raciocínio inverso ao do exercício 1. Um aluno 
esperto numa prova dessa olharia para a questão 1 e veria que a 
probabilidade de 2/3 equivale à chance de 2 para 1 (para a derrota de ambos 
os carros, nesse caso). Mas o raciocínio seria: se em 3 há duas derrotas, 
sobra 1 para a vitória: 2(derrotas) para 1(vitória), 2 para 1.


4) 9 para 7 equivale a uma probabilidade de 9/16 para a vitória: 56,25%

Abraço a todos,
João Luís.


- Original Message - 
From: Jorge Luis Rodrigues e Silva Luis [EMAIL PROTECTED]

To: obm-l@mat.puc-rio.br
Sent: Thursday, February 23, 2006 9:49 AM
Subject: [obm-l] PROBABILIDADES  CHANCE!


Um atleta atribui uma chance de 2 para 1, mas não de 3 para 1, de que ele 
derrotará seu companheiro em uma queda de braço. O que é que isto nos diz 
sobre a probabilidade que ele atribui à sua vitória?


Se a chance de um time de futebol americano de uma faculdade ganhar o 
próximo jogo é melhor do que 5 para 4, o que se pode dizer quanto à 
probabilidade de vencer o próximo jogo?


Há dois Porsches em uma corrida, e um repórter acha que as chances contra 
suas vitórias são de 4 para 1 e 5 para 1 respectivamente. Para ser 
consistente, que chance ele deve atribuir ao evento de nenhum dos dois 
carros ganhar?


Qual a probabilidade de Piquet vencer determinada corrida se, segundo os 
técnicos de sua escuderia as suas chances são de 9 para 7?


Abraços!

_
Copa 2006: Sabe como se diz 'pênalti' em alemão? Clique aqui! 
http://copa.br.msn.com/extra/dicionario/l-z/


=
Instruções para entrar na lista, sair da lista e usar a lista em
http://www.mat.puc-rio.br/~nicolau/olimp/obm-l.html
= 



=
Instruções para entrar na lista, sair da lista e usar a lista em
http://www.mat.puc-rio.br/~nicolau/olimp/obm-l.html
=


[obm-l] PROBABILIDADES!

2005-12-05 Por tôpico Jorge Luis Rodrigues e Silva Luis
Valeu, Leonardo! Quanto aos quadrados mágicos, a resposta do colega Bruno 
tem grandes probabilidades de estar correta...


Um quadrado de lado 3 é dividido em 9 quadrados de lado unitário, formando 
um quadriculado. Cada quadrado unitário é pintado de azul ou vermelho. Cada 
cor tem probabilidade 1/2 de ser escolhida e a cor de cada quadrado é 
escolhida independentemente das demais. Qual a probabilidade de obtermos, 
após colorirmos todos os quadrados unitários, um quadrado de lado 2 pintado 
inteiramente de uma mesma cor?


A probabilidade de a equipe A vencer qualquer jogo é 1/2. A e B disputam 
entre si um torneio. A primeira equipe que conseguir vencer dois jogos em 
seguida ou um total de três jogos vence o torneio. Determinar o número 
esperado de jogos do torneio.


Lançam-se dois dados honestos. Qual a probabilidade de que a diferença em 
módulo das faces seja menor do que 2?


A propósito, se a probabilidade que tenho de perder meu irmão daqui a 30 
anos é de 6/11, porque a probabilidade de ocorrer o inverso não é 5/11?


Um abraço a todos!

_
MSN Messenger: converse online com seus amigos .  
http://messenger.msn.com.br


=
Instruções para entrar na lista, sair da lista e usar a lista em
http://www.mat.puc-rio.br/~nicolau/olimp/obm-l.html
=


[obm-l] PROBABILIDADES!

2005-12-05 Por tôpico eritotutor

Lançam-se dois dados honestos. Qual a probabilidade de que a diferença em
 módulo das faces seja menor do que 2?
16/36 

Um quadrado de lado 3 é dividido em 9 quadrados de lado unitário, formando
 um quadriculado. Cada quadrado unitário é pintado de azul ou vermelho. Cada 
 cor tem probabilidade 1/2 de ser escolhida e a cor de cada quadrado é 
 escolhida independentemente das demais. Qual a probabilidade de obtermos, 
 após colorirmos todos os quadrados unitários, um quadrado de lado 2 pintado 
 inteiramente de uma mesma cor?
Existem 4 quadrados de lado 2.
p = 4*2*[(1/2)^4]

[]s


 Valeu, Leonardo! Quanto aos quadrados mágicos, a resposta do colega Bruno 
 tem grandes probabilidades de estar correta...
 
 Um quadrado de lado 3 é dividido em 9 quadrados de lado unitário, formando 
 um quadriculado. Cada quadrado unitário é pintado de azul ou vermelho. Cada 
 cor tem probabilidade 1/2 de ser escolhida e a cor de cada quadrado é 
 escolhida independentemente das demais. Qual a probabilidade de obtermos, 
 após colorirmos todos os quadrados unitários, um quadrado de lado 2 pintado 
 inteiramente de uma mesma cor?
 
 A probabilidade de a equipe A vencer qualquer jogo é 1/2. A e B disputam 
 entre si um torneio. A primeira equipe que conseguir vencer dois jogos em 
 seguida ou um total de três jogos vence o torneio. Determinar o número 
 esperado de jogos do torneio.
 
 Lançam-se dois dados honestos. Qual a probabilidade de que a diferença em 
 módulo das faces seja menor do que 2?
 
 A propósito, se a probabilidade que tenho de perder meu irmão daqui a 30 
 anos é de 6/11, porque a probabilidade de ocorrer o inverso não é 5/11?
 
 Um abraço a todos!
 
 _
 MSN Messenger: converse online com seus amigos . 
 http://messenger.msn.com.br
 
 =
 Instruções para entrar na lista, sair da lista e usar a lista em
 http://www.mat.puc-rio.br/~nicolau/olimp/obm-l.html
 =


RES: [obm-l] Probabilidades

2005-08-09 Por tôpico Artur Costa Steiner



A primeira equacao 
so vale se os eventos forem disjuntos 2 a 2 e eh uma consequencia imediata da 
segunda, poistodas as interseccoes tem entao probabilidade 
nula.

Uma forma de provar 
2 eh de fato por inducao. Uma outra forma eh observando que, ao computar 
P(A1)+P(A2)+...+P(An), voce contou diversas vezes as intersecoes dos eventos 
(que podem ser vistos como conjuntos). A parcela S ij)P(Ai interseção Aj), 
que eh deduzida, considera as probabilidades das interseccoesdos eventos 2 
a 2. Mas ao fazer isto, vc tirou dermais, pois retirou varias vezes as 
intersecoes 3 a 3, 4 a 4, etc. A parcela .S ijk)P(Ai inters Aj inters 
Ak) agora "devolve" estas probailidades, mas devolvedemais, pois agrega 
varias vezes as 4 a 4 , etc. Daih vc tem que continuar o processo ate a parcela 
final P(A1 inters ... inters An), que representa a probailidade das interseccoes 
de todos os eventos. Esta parcela podera ter sinal positivo ou negativo. 
Positivo se houver um numero impar de eventos e negativo se houver um numero 
par.

Artur


-Mensagem original-De: 
[EMAIL PROTECTED] [mailto:[EMAIL PROTECTED]Em nome de 
Luiz ViolaEnviada em: sábado, 6 de agosto de 2005 
10:15Para: Lista de matemática da PUCAssunto: [obm-l] 
Probabilidades

  Será que alguém me ajuda com esses dois problemas 
  de probabilidades? Sei que pode parecer trivial para vocês mas sou da área de 
  economia e não tenho tanta intimidade assim com a matemática.
  
  1) Provarque:
  
  P(A1 U A2 U ... U An) = 
  P(A1)+P(A2)+...+P(An)
  
  
  2) Provar que
  
   P(A1 U A2 U ... U An) = (S 
  i)P(Ai) - (S ij)P(Ai interseção Aj) + (S ijk)P(Ai inters Aj inters 
  Ak) - ... +- P(A1 inters ... inters An)
  
   S i é o somatório com índice 
  i
  
  
  Acho que essa última prova surge por indução não? 
  Desculpem-mese a notação ficou bagunçada. Foi a melhor maneira que eu 
  consegui para escrever as igualdades...
  
  Abraços
  


[obm-l] Probabilidades

2005-08-06 Por tôpico Luiz Viola



Será que alguém me ajuda com esses dois problemas 
de probabilidades? Sei que pode parecer trivial para vocês mas sou da área de 
economia e não tenho tanta intimidade assim com a matemática.

1) Provarque:

P(A1 U A2 U ... U An) = 
P(A1)+P(A2)+...+P(An)


2) Provar que

 P(A1 U A2 U ... U An) = (S 
i)P(Ai) - (S ij)P(Ai interseção Aj) + (S ijk)P(Ai inters Aj inters 
Ak) - ... +- P(A1 inters ... inters An)

 S i é o somatório com índice 
i


Acho que essa última prova surge por indução não? 
Desculpem-mese a notação ficou bagunçada. Foi a melhor maneira que eu 
consegui para escrever as igualdades...

Abraços



Re: [obm-l] Probabilidades

2004-08-16 Por tôpico Grupo de Matematica

 Muito obrigado pelas explicações, não poderiam ser mais esclarecedoras. Boa sorte a Araray Velho no desenvolvimento de seu trabalho.

 Um abraço,

 Rafael Lima




On Fri Aug 13 12:48 , Artur Costa Steiner [EMAIL PROTECTED] sent:



Nota-se, experimentalmente, que a natureza traduzida para a linguagem
matemᴩca nem sempre manifesta uma express㯠precisa: as chances de um
determinado evento ocorrer 頤e 1/10, mas n㯠頣erto que em 10 tentativas
o evento ocorrerá µma vez (é °ossí¶¥l que ocorra mais de uma vez ou, mais
provavelmente, n㯠ocorra).

Sempre que usamos matematica para descrever algum fenomeno da vida real,
seja ele fisico, biologico, economico, social, etc, fazemos necessariamente
algumas simplificacoes. Quase nunca conhecemos de forma absolutamente
precisa as relacoes de causa e efeito entre as diversas variaveis que
interferem no fenomeno. Segue-se daih que todo modelo matematico de alguma
situacao da vida real eh uma simplificaco da realidade. Naum por causa da
matematica. mas sim pela nossa impossibiliddae de representar extamente o
fenomeno em estudo. Por acusa de nossa ignorancia, palavra que, aqui, naum
tem qualquer sentido pejorativo, mas significa simplesmente que naum
conhecemos tudo 
No caso de probabilidades, quando saimos da definicao axiomatica da
matematica e entramos em processos reais, temos necessariamente que nos
adaptar ao fenomeno em analise. Se um fenomeno eh aleatorio, eh porque naum
temos conhecimento total sobre ele. Naum conseguimos prever o "output" para
um dado "input", embora possamos reconhecer uma certa regularidade que
diferencia fenomenos aleatorios daqueles erraticos. 
Isto naum me parece invalidar a aplicacao da teoria de probabilidades e a da
estatistica. Apesar das limitacoes, muitos fenomenos da vida real sao
analisados com bons resultados utilizando-se tais ciencias. Um dels eh a
producao de energia eletrica em nosso pais.


É bem verdade, entretanto, que a estimativa se torna mais acurada quando,
em uma repeti磯 maior de tentativas, a raz㯠entre ocorrꮣias e testes
se aproxima da previs㯠num鲩ca. Assim acontece com as experiꮣias
adotadas no mé´¯do cientí¦©co, seja em qual for a á²¥a de conhecimento.
 Basicamente, interessa saber o por quê ¤e para chances estimadas muito
pequenas, o n? de tentativas para que se aproxime da previs㯠頴㯼/font
maior, a ponto de se tornar impossí¶¥l na prá´©ca, testemunhar tal evento
(ou remoto de maneira tal que jamais será ¶isto). O que significa isto?
Poderia ser uma incoerꮣia matemᴩca ou 頡plica磯 indevida?

Naum me parece que seja uma incoerencia matematica. A probabilidade,
enquanto definida axiomaticamente, ateh reforca o que vc disse. Se a
probabilidade de um experimento ter sucesso eh eh p e realizamos repeticoes
independentes dele, entao valor espertado do numero de realizoes necessarias
para se ter um primeiro sucesso eh 1/p, que cresce aa medida em que p - 0.
Tambem naum me parece aplicacao indevida, mas sim uma constatacao de como eh
nosso mundo real. A matematica em muito contribui para melhor trabalharmos
nosso mundo, mas nel naum podemos demonstrar fatos como aqueles que
encontramos nos livros de Matematica. Muito temos que assumir com base na
experiꮣia. Ninguem duvida das lei da gravidade, mas naum hah uma prova
matematica de que ela seja verdaeira. Uma coisa eh escrever num livro de
Analise "seja f uma funcao diferenciavel. Entao". Outra, na vida real,
quando f eh uma funcao que meç¡ algo concreto, como o custo de operacao de
um sistema em funcao de sua carga, ter certeza de que f eh diferenciavel.
Muitas vezes, nao temos f definida de forma que possamos fazer tal tipo de
afirmacao.

Espero naum ter me afastado muito de seus pontos
Artur 



OPEN Internet
@ Primeiro provedor do DF com anti-ví²µs no servidor de e-mails @


=
Instruçµ¥s para entrar na lista, sair da lista e usar a lista em
http://www.mat.puc-rio.br/~nicolau/olimp/obm-l.html
=

Introducing Spymac MailPro: http://www.spymac.com/mailpro/
Instruções para entrar na lista, sair da lista e usar a lista em
http://www.mat.puc-rio.br/~nicolau/olimp/obm-l.html


RE: [obm-l] Probabilidades

2004-08-13 Por tôpico Paulo Santa Rita
Ola,
Na *Matematica,  algo em si incoerente,  nao tem interesse e nao existe. Uma 
aplicacao, claramente, pode ser incoerente, pois a linguagem que se esta 
usando pode nao ser a modelagem adeguada. Procure informacoes sobre a LEI 
DOS GRANDES NUMEROS (Teorema de Bernoulli ) e a desigualdade de Chebishev.  
Estes temas se referem diretamente as sua observacoes.

*Na Matematica Classica, rotineira e habitual. Um Sistema formal pode ser 
inconsistente e nem por isso ser trivial ( Newton Costa ). Mas a 
paraconsistencia, evidentemente, nao se aplica ao seu universo de interesse.

Um Abraco
Paulo Santa Rita
6,1038,130804
From: Grupo de Matematica [EMAIL PROTECTED]
Reply-To: [EMAIL PROTECTED]
To: [EMAIL PROTECTED]
Subject: [obm-l] Probabilidades
Date: Thu, 12 Aug 2004 19:32:00 -0600
  Nota-se, experimentalmente, que a natureza traduzida para a linguagem 
matemática nem sempre manifesta uma expressão precisa: as chances de um 
determinado evento ocorrer é de 1/10, mas não é certo que em 10 tentativas 
o evento ocorrerá uma vez (é possível que ocorra mais de uma vez ou, mais 
provavelmente, não ocorra). É bem verdade, entretanto, que a estimativa se 
torna mais acurada quando, em uma repetição maior de tentativas, a razão 
entre ocorrências e testes se aproxima da previsão numérica. Assim 
acontece com as experiências adotadas no método científico, seja em qual 
for a área de conhecimento.
  Basicamente, interessa saber o por quê de para chances estimadas muito 
pequenas, o número de tentativas para que se aproxime da previsão é tão 
maior, a ponto de se tornar impossível na prática, testemunhar tal evento 
(ou remoto de maneira tal que jamais será visto). O que significa isto? 
Poderia ser uma incoerência matemática ou é aplicação indevida?
  Por favor, em sua aguardada resposta, considere a Matemática como uma 
linguagem criada para conduzir estudos do universo, independentemente de 
ela ter outras conotações.

  Grato,
  Rafael Lima (pelo Grupo de Matemática)
  PS.: Aproveito a oportunidade para apresentar à lista o grupo e 
antecipar meu pedido de desculpa se fugi da proposta da obm-l.
_
MSN Hotmail, o maior webmail do Brasil.  http://www.hotmail.com
---BeginMessage---


 Nota-se, experimentalmente, quea natureza traduzida para a linguagem matemática nem sempre manifesta uma expressão precisa: as chances de um determinado evento ocorrer é de 1/10, mas não é certo que em 10 tentativas o evento ocorrerá uma vez (é possível que ocorra mais de uma vez ou, mais provavelmente, não ocorra). É bem verdade, entretanto, que a estimativa se torna mais acurada quando, em uma repetição maior de tentativas, a razão entreocorrências etestesse aproxima da previsão numérica. Assimacontece com asexperiências adotadas no método científico, seja em qual for a área de conhecimento.
 Basicamente, interessa saber oporquê depara chances estimadasmuito pequenas, o número de tentativas para que se aproxime da previsão é tão maior, a ponto de se tornarimpossívelna prática, testemunhar tal evento (ou remotode maneira talque jamais será visto). O que significa isto? Poderia seruma incoerência matemática ou éaplicação indevida?
 Por favor, emsua aguardadaresposta, considere a Matemática como uma linguagemcriada para conduzir estudos do universo,independentemente deelater outras conotações.

 Grato,

 Rafael Lima (pelo Grupo de Matemática)

 PS.: Aproveito a oportunidadeparaapresentar à lista o grupo e antecipar meu pedido de desculpa se fugi da proposta da obm-l.
 Introducing Spymac MailPro: http://www.spymac.com/mailpro/
Instruções para entrar na lista, sair da lista e usar a lista em
http://www.mat.puc-rio.br/~nicolau/olimp/obm-l.html

---End Message---


Re: [obm-l] Probabilidades

2004-08-13 Por tôpico Artur Costa Steiner

Nota-se, experimentalmente, que a natureza traduzida para a linguagem
matemática nem sempre manifesta uma expressão precisa: as chances de um
determinado evento ocorrer é de 1/10, mas não é certo que em 10 tentativas
o evento ocorrerá uma vez (é possível que ocorra mais de uma vez ou, mais
provavelmente, não ocorra).

Sempre que usamos matematica para descrever algum fenomeno da vida real,
seja ele fisico, biologico, economico, social, etc, fazemos necessariamente
algumas simplificacoes. Quase nunca conhecemos de forma absolutamente
precisa as relacoes de causa e efeito entre as diversas variaveis que
interferem no fenomeno. Segue-se daih que todo modelo matematico de alguma
situacao da vida real eh uma simplificaco da realidade. Naum por causa da
matematica. mas sim pela nossa impossibiliddae de representar extamente o
fenomeno em estudo. Por acusa de nossa ignorancia, palavra que, aqui, naum
tem qualquer sentido pejorativo, mas significa simplesmente que naum
conhecemos tudo   
No caso de probabilidades, quando saimos da definicao axiomatica da
matematica e entramos em processos reais, temos necessariamente que nos
adaptar ao fenomeno em analise. Se um fenomeno eh aleatorio, eh porque naum
temos conhecimento total sobre ele. Naum conseguimos prever o output para
um dado input, embora possamos reconhecer uma certa regularidade que
diferencia fenomenos aleatorios daqueles erraticos. 
Isto naum me parece invalidar a aplicacao da teoria de probabilidades e a da
estatistica. Apesar das limitacoes, muitos fenomenos da vida real sao
analisados com bons resultados utilizando-se tais ciencias. Um dels eh a
producao de energia eletrica em nosso pais.


É bem verdade, entretanto, que a estimativa se torna mais acurada quando,
em uma repetição maior de tentativas, a razão entre ocorrências e testes
se aproxima da previsão numérica. Assim acontece com as experiências
adotadas no método científico, seja em qual for a área de conhecimento.
   Basicamente, interessa saber o por quê de para chances estimadas muito
pequenas, o número de tentativas para que se aproxime da previsão é tão
maior, a ponto de se tornar impossível na prática, testemunhar tal evento
(ou remoto de maneira tal que jamais será visto). O que significa isto?
Poderia ser uma incoerência matemática ou é aplicação indevida?

Naum me parece que seja uma incoerencia matematica. A probabilidade,
enquanto definida axiomaticamente, ateh reforca o que vc disse. Se a
probabilidade de um experimento ter sucesso eh eh p e realizamos repeticoes
independentes dele, entao valor espertado do numero de realizoes necessarias
para se ter um primeiro sucesso eh 1/p, que cresce aa medida em que p - 0.
Tambem naum me parece aplicacao indevida, mas sim uma constatacao de como eh
nosso mundo real. A matematica em muito contribui para melhor trabalharmos
nosso mundo, mas nel naum podemos demonstrar fatos como aqueles que
encontramos nos livros de Matematica. Muito temos que assumir com base na
experiência. Ninguem duvida das lei da gravidade, mas naum hah uma prova
matematica de que ela seja verdaeira. Uma coisa eh escrever num livro de
Analise seja f uma funcao diferenciavel. Entao. Outra, na vida real,
quando f eh uma funcao que meça algo concreto, como o custo de operacao de
um sistema em funcao de sua carga, ter certeza de que f eh diferenciavel.
Muitas vezes, nao temos f definida de forma que possamos fazer tal tipo de
afirmacao.

Espero naum ter me afastado muito de seus pontos
Artur 



OPEN Internet
@ Primeiro provedor do DF com anti-vírus no servidor de e-mails @


=
Instruções para entrar na lista, sair da lista e usar a lista em
http://www.mat.puc-rio.br/~nicolau/olimp/obm-l.html
=


[obm-l] Probabilidades

2004-08-12 Por tôpico Grupo de Matematica


 Nota-se, experimentalmente, quea natureza traduzida para a linguagem matemática nem sempre manifesta uma expressão precisa: as chances de um determinado evento ocorrer é de 1/10, mas não é certo que em 10 tentativas o evento ocorrerá uma vez (é possível que ocorra mais de uma vez ou, mais provavelmente, não ocorra). É bem verdade, entretanto, que a estimativa se torna mais acurada quando, em uma repetição maior de tentativas, a razão entreocorrências etestesse aproxima da previsão numérica. Assimacontece com asexperiências adotadas no método científico, seja em qual for a área de conhecimento.
 Basicamente, interessa saber oporquê depara chances estimadasmuito pequenas, o número de tentativas para que se aproxime da previsão é tão maior, a ponto de se tornarimpossívelna prática, testemunhar tal evento (ou remotode maneira talque jamais será visto). O que significa isto? Poderia seruma incoerência matemática ou éaplicação indevida?
 Por favor, emsua aguardadaresposta, considere a Matemática como uma linguagemcriada para conduzir estudos do universo,independentemente deelater outras conotações.

 Grato,

 Rafael Lima (pelo Grupo de Matemática)

 PS.: Aproveito a oportunidadeparaapresentar à lista o grupo e antecipar meu pedido de desculpa se fugi da proposta da obm-l.
 Introducing Spymac MailPro: http://www.spymac.com/mailpro/
Instruções para entrar na lista, sair da lista e usar a lista em
http://www.mat.puc-rio.br/~nicolau/olimp/obm-l.html


Re: [math] [obm-l] Probabilidades

2004-08-12 Por tôpico Araray Velho
Olá !

Tenho me ocupado com o desenvolvimento de um novo tipo de probabilidade que
é capaz de dar respostas mais lógicas para essas perguntas. A probabilidade
convencional é funcional para eventos que não pertençam ao Universo Físico,
pois o mesmo possui tantas variáveis que qualquer estimativa estaria
impregnada de incertezas e imprecisões diversas. No entanto, ela é muito boa
para eventos abstratos, como a própria matemática. Como disse, a
probabilidade na qual eu tenho trabalhado se refere aos eventos do Universo
Físico.

Não sei se fui muito útil, mas, enfim, era isso que eu gostaria de
acrescentar.

Abraços.

Araray Velho


- Original Message - 
From: Grupo de Matematica
To: [EMAIL PROTECTED]
Sent: Thursday, August 12, 2004 10:32 PM
Subject: [math] [obm-l] Probabilidades


   Nota-se, experimentalmente, que a natureza traduzida para a linguagem
matemática nem sempre manifesta uma expressão precisa: as chances de um
determinado evento ocorrer é de 1/10, mas não é certo que em 10 tentativas o
evento ocorrerá uma vez (é possível que ocorra mais de uma vez ou, mais
provavelmente, não ocorra). É bem verdade, entretanto, que a estimativa se
torna mais acurada quando, em uma repetição maior de tentativas, a razão
entre ocorrências e testes se aproxima da previsão numérica. Assim acontece
com as experiências adotadas no método científico, seja em qual for a área
de conhecimento.
   Basicamente, interessa saber o por quê de para chances estimadas muito
pequenas, o número de tentativas para que se aproxime da previsão é tão
maior, a ponto de se tornar impossível na prática, testemunhar tal evento
(ou remoto de maneira tal que jamais será visto). O que significa isto?
Poderia ser uma incoerência matemática ou é aplicação indevida?
   Por favor, em sua aguardada resposta, considere a Matemática como uma
linguagem criada para conduzir estudos do universo, independentemente de ela
ter outras conotações.

   Grato,

   Rafael Lima (pelo Grupo de Matemática)

   PS.: Aproveito a oportunidade para apresentar à lista o grupo e antecipar
meu pedido de desculpa se fugi da proposta da obm-l.

 Introducing Spymac MailPro: http://www.spymac.com/mailpro/
Instruções para entrar na lista, sair da lista e
usar a lista em http://www.mat.puc-rio.br/~nicolau/olimp/obm-l.html


=
Instruções para entrar na lista, sair da lista e usar a lista em
http://www.mat.puc-rio.br/~nicolau/olimp/obm-l.html
=


Re: [obm-l] Probabilidades

2004-08-12 Por tôpico Ricardo Bittencourt
Grupo de Matematica wrote:
   Nota-se, experimentalmente, que a natureza traduzida para a linguagem 
matemática nem sempre manifesta uma expressão precisa: as chances de um 
determinado evento ocorrer é de 1/10, mas não é certo que em 10 
tentativas o evento ocorrerá uma vez (é possível que ocorra mais de uma 
vez ou, mais provavelmente, não ocorra). 

A chance de que o evento ocorra uma vez na primeira
rodada e nenhuma nas outras 9 é de (1/10).(9/10)^9. Da mesma
maneira, a chance de que ocorra apenas na segunda rodada
e em nenhuma outra também é (1/10).(9/10)^9. Somando tudo,
a chance de que o evento ocorra uma vez só em dez experimentos
é de 10.(1/10).(9/10)^9 = 38% aproximadamente.

Ricardo Bittencourt   http://www.mundobizarro.tk
[EMAIL PROTECTED]   tenki ga ii kara sanpo shimashou
-- União contra o forward - crie suas proprias piadas --
=
Instruções para entrar na lista, sair da lista e usar a lista em
http://www.mat.puc-rio.br/~nicolau/olimp/obm-l.html
=


[obm-l] Probabilidades

2004-01-17 Por tôpico Everton A. Ramos (www.bs2.com.br)
Eu possuo 20 letras, quero combinar as mesmas em grupos de 5, de forma que
não existam grupos formados pelas mesmas letras.

Ou seja, o grupo ABCDE e o grupo ABCED devem ser considerados como iguais,
qual a forma para fazer tal cálculo?

Obrigado


Everton A. Ramos
Desenvolvimento de Sistemas
(44) 8801-0186 / (27) 8111-8652
[EMAIL PROTECTED]


=
Instruções para entrar na lista, sair da lista e usar a lista em
http://www.mat.puc-rio.br/~nicolau/olimp/obm-l.html
=


Re: [obm-l] Probabilidades

2004-01-17 Por tôpico Faelccmm
C(20,5) = 20 ! / 5! (20 - 5) !
C(20,5) = 20 ! / 5! * 15!
C(20,5) = 20*19*18*17*16*15! / 5! * 15!
C(20,5) = 20*19*18*17*16 / 5*4*3*2*1
C(20,5) = 15504

Logo ha 15504 grupos de 5 letras, de forma que
não existam grupos formados pelas mesmas letras.

Ps: Se errei em algo me corrijam.




Em uma mensagem de 17/1/2004 20:59:36 Hor. de verão leste da Am. Sul, [EMAIL PROTECTED] escreveu:


Eu possuo 20 letras, quero combinar as mesmas em grupos de 5, de forma que
não existam grupos formados pelas mesmas letras.

Ou seja, o grupo ABCDE e o grupo ABCED devem ser considerados como iguais,
qual a forma para fazer tal cálculo?

Obrigado


Everton A. Ramos
Desenvolvimento de Sistemas
(44) 8801-0186 / (27) 8111-8652
[EMAIL PROTECTED]






Re: [obm-l] Probabilidades

2004-01-05 Por tôpico Flavio
houve um erro de "do" a mais
a pergunta eh

Qual a probabilidade do ponto A ser maior do queo ponto B??

OI pessoal,

Acompanho a lista a pouco tempo e a acho muito interessante
Esses dias me apareceu o seguinte problema

possuo 2 variaveis distintas e de mesma caracteristica (tempo)
essas 2 variaveis possuem uma curva probabilistica de distribuicao normal com desvio padrao

ex. variavel a - media " a/ "e desvio padrao "da "
 variavelb - media " b/ "e desvio padrao "db "

a partir da curva probabilistca normal, seleciona-se um ponto totalmente aleatorio pertencente a curva "a" e outro ponto da curva "b" chamados de "A" e "B" respectivamente

o que eu gostaria de saber eh:

Qual a probabilidade do ponto A ser maior do queo do ponto B??

estou tendo um serios problemas com as integrais


desde ja agradeco

Abracos
Flavio


Central anti-spam do Yahoo! Mail: com dicas, dúvidas e curiosidades!